Kaplan Conlaw Flashcards
1
Q
- During the final days of his presidency, many of the President’s Cabinet members began leaking rather embarrassing stories about the President to the press. In one such story, which appeared in a newspaper, an unidentified Cabinet member was quoted as saying that the President pardoned the owner of a professional baseball team, for a previous felony conviction because the owner promised to give the President a lifetime pass to all future games. Upset by these news “leaks,” the President, in his last official act, issued an executive order banning all executive employees from having conversations with members of the press unless prior permission had been obtained from an administrative supervisor. The executive order called for the dismissal of any employee willfully in violation of that order.After the President left office, the press made repeated references to the executive order and derisively referred to it as the President’s legacy. Nonetheless, after the new President took office he promised to abide by the order. Thereafter one of the Vice President’s senior advisers, leaked a story to a magazine stating that the Vice President told staff members that the only reason he visited a city following an earthquake was because he wanted to play golf at a nearby golf course. The magazine printed the story and named the senior adviser as the source. In compliance with civil service regulations, a hearing was conducted and the senior adviser was found to have violated the executive order barring unauthorized conununications with print media. Subsequently, the senior adviser was summarily dismissed from his executive staff position.The senior adviser brings an appropriate action seeking reinstatement and challenges the constitutionality of the executive order. If this case reaches the U.S. Supreme Court, the justices should rule the executive order is(A) constitutional, because governmental employment is a privilege and not a right.(B) constitutional, because the President has plenary power to set forth employment regulations for executive department employees.(C) unconstitutional, because Congress, not the President, has authority to regulate the terms and conditions of federal employment.(D) unconstitutional, because the President cannot broadly limit all executive employees’ freedom of speech and association.
A
- (D) Choice (A) is incorrect. As a general rule, the President may terminate, without Congress’s approval, those appointees, like his advisers. However, the President may not violate the federal Constitution in doing so. In this instance, the President’s order requiring his appointees to obtain prior permission to speak probably violates the FirstAmendment. Choice (B) is incorrect. The President may not draft regulations that violate the constitutional rights of the employees. After all, the President is no mere employer; he or she is a state actor, and the Constitution protects people, including governmental employees, from constitutional violations by state actors in their official capacity. Choice (C) is too narrow. The President has the authority to regulate the terms and conditions of federal employment for certain employees, especially those, like his advisers who are directly and solely responsible to the President. While the President has the power both to issue executive orders and to remove purely executive officers appointed by him without being subject to any Congressional limitation, the scope of these powers must not contravene federal constitutional limitations, such as the First Amendment rights of freedom of speech and association. The executive order banning all executive employees from having conversations with members of the press unless prior permission has been obtained from an administrative supervisor probably violates the First Amendment as an overbroad prior restraint limiting otherwise protected speech. The executive order is, therefore, unconstitutional. Therefore, choice (D) is the correct answer.
2
Q
- A devastating earthquake struck a foreign country. The earthquake destroyed hundreds of homes and buildings. Thousands of residents were killed, injured, and left homeless by the earthquake. The leader of the foreign country sought the assistance of the United States in providing emergency aid.The President initially wanted to send troops from a west coast military base but learned that they were out on training maneuvers. As a result, the President issued an executive order directing members of a federal agency under the supervision of the Immigration and Naturalization Service, to travel to the foreign country to engage in emergency rescue operations. This federal agency’s main function is to patrol the borders to prevent illegal aliens from entering the United States.A member of this federal agency refused to comply with the executive order and assist in the relief effort. He claimed that participating in such operations in a foreign country is beyond the scope of his job responsibilities. He brought suit in federal court challenging the validity of the President’s executive order. Assuming the member of the federal agency has adequate standing, the court should rule the executive order(A) valid, because as commander-in-chief, the President has unfettered authority to require federal employees to participate in an international relief effort.(B) valid, because employees of an executive agency can be required to perform such emergency functions.(C) invalid, because the executive order is beyond the scope of presidential power absent congressional authorization.(D) invalid, because the director of the INS, not the President, has the power to issue directives to federal agencies under its supervision.
A
- (B) The Constitution gives the President broad authority with respect to foreign affairs. Article II, Section 2 explicitly enumerates a number of powers given him in this area (e.g., commander-in-chief power, treaty-making power, and the right to appoint am bassadors). However, the President’s power over foreign affairs is not plenary or absolute. Rather, the President and Congress have concurrent powers over foreign affairs. To be sure, Congress is given war power authority and treaties must be ratified by two-thirds of the Senate. In this regard, choice (A) is not the best answer for two reasons. First, it is doubtful that the President would have authority as commander- in-chief to mobilize nonmilitary federal employees in this manner. Second, the President’s authority to direct foreign affairs is not “unfettered” or absolute. Choice (B) would be a better answer, since the President as Chief Executive would probably have authority to direct executive employees to engage in such an international relief effort. Choice (C) is incorrect. This is a little too strong. To avoid violating the principle of the separation of powers, it is wise for the President to seek congressional approval for an executive order. However, it is not necessary in all instances that the President seek approval from Congress before issuing his executive order. This is especially true for issues that are related to foreign affairs. Choice (D) is a bit too narrow. The INS is encompassed under the Department of Homeland Security, which, in turn, is housed under the Executive Branch.
3
Q
- Congress enacted a statute wherein the government would purchase and operate all of the nation’s airlines.In all likelihood, Congress’s power to enact this legislation will derive from(A) its power to tax and provide for the general welfare.(B) its power to raise and support an army and declare war.(C) its power to regulate commerce.(D) its power to make laws regarding territory and other property belonging to the United States.
A
- (C) Since Congress has the power to regulate interstate commerce under Article I, Section 8, choice (C) is the best answer. Choice (B) is incorrect because the facts do not state that the United States is at war. Similarly, you may not assume that the airline takeover was a wartime or war-related act. Choice (A) is incorrect. In order to purchase aLL of these airlines, it’s possible that Congress will have to levy higher taxes on Americans. But the question asks under what power Congress may purchase the airlines, not how and if it will pay for them. And Congress can use its commerce clause powers to purchase the airlines. Finally, choice (D) is incorrect. The airlines didn’t belong to the United States at the time of purchase.
4
Q
- Congress enacted a statute designed to make the stabilization of rents more effective throughout the United States. The statute will be applicable to all leasehold contracts hereafter entered into between landlords and tenants involving residential and commercial properties. The new federal statute is intended to protect tenants from spiraling rents and “profiteering” by landlords without curtailing the supply of rental units or depriving landlords of substantial constitutional rights.The new federal statute would be most clearly constitutional as applied to(A) the rental of a state-owned office building by a state to a privately owned business.(B) the rental of residential property located in the District of Columbia by an individual landlord to a family of six.(C) the rental of an apartment located by a real estate company to an individual tenant.(D) the rental of an office building to a city by an individual landlord.
A
- (B) ArticLe IV, Section 3, Clause 2 gives Congress power “to dispose of and make all needfuL ruLes and regulations respecting” lands of the United States. Since the District of Columbia is under federal jurisdiction, choice (B) is the best answer. Choice (A) is incorrect. ArticLe IV, Section 3, Clause 2 gives Congress power “to dispose of and make all needful rules and regulations respecting” Lands of the United States. For purposes of this clause, a state does not fall within Congress’s federal jurisdiction. Choice (C) is incorrect. Article IV, Section 3, CLause 2 gives Congress power “to dispose of and make all needful rules and regulations respecting” Lands of the United States. For purposes of this clause, a state does not faLL within Congress’s federaL jurisdiction. Choice (D) is not the best choice. Article IV, Section 3, Clause 2 gives Congress power “to dispose of and make all needful rules and regulations respecting” Lands of the United States. For purposes of this clause, a city does not faLl within Congress’s federal jurisdiction.
5
Q
- Congress has recently enacted a statute designed to prevent price gouging of gasoline by private retail sellers. To help administer the new statute Congress created a seven-member agency, four members of which were appointed by the President, two by the Senate, and one by the Speaker of the House. The agency was authorized to issue rules interpreting regulations prescribed under existing federal law. It was also authorized to issue “cease and desist” orders after hearings against private retail sellers of gasoline who “intentionally engaged in price gouging.” In this regard, violations of the cease and desist orders were made punishable by fine and/or imprisonment.Which of the following is the strongest constitutional argument against the authority of this agency?(A) Congress may not delegate to the agency power to make interpretations of existing federal laws.(B) Congress may not delegate to the agency power to make “cease and desist” orders, the violation of which is punishable by imprisonment.(C) Congress may not delegate enforcement of a federal law in any governmental body other than the U.S. Attorney General or the courts.(D) Congress may not retain the power to appoint even a minority of members of such an agency.
A
- (D) Article II, Section 2 specifies that the President shall appoint “with the advice and consent of the Senate,” all “Ambassadors, other public Ministers and Consuls, judges of the Supreme Court, and all other Officers of the United States.” Note that although Congress may appoint officials to exercise such investigative power as it might delegate to one of its own committees, it maynotappoint members oran agency or commission with administrative powers. Such persons are deemed “officers of the United States” and must be appointed pursuant to Article II, Section 2 [Buckleyv. Valeo, 424 U.S. 1 (1976)]. Choice (A) is incorrect. In the fact situation, Congress authorized the agency to do more than interpret the laws. Congress authorized it to punish violators. But such authorization goes beyond what Congress is permitted to do. Under Article II, Section 2, the President shall appoint “with the advice and consent of the Senate,” all “Ambassadors, other public Ministers and ConsuLs, judges of the Supreme Court, and all other Officers of the United States.” In other words, only the President may appoint those with administrative powers to punish law violators. Choice (B) is a good choice, but it’s not the best choice. The problem is larger than one of “cease and desist” orders. For Congress lacks the authority to appoint members to an agency that has any administrative powers, including any powers to punish law violators. Congress would do no better if it had substituted the right to issue cease and desist orders with some other form of punishment. Only the President has authority to appoint members who may exercise administrative power [Buckleyv. Valeo, 424 U.S. 1 (1976)], construing Article II, Section 2. Finally, choice (C) is incorrect. The Attorney General certainly has enforcement powers, given that he is an officer of the Executive Branch. So, too, the courts may issue cease and desist orders. Congress also has the authority under Article I to create agencies that may enforce the law. Congress may not, however, appoint members to these agencies; only the President may do so [Buckleyv. Valeo, 424 U.S. 1 (1976)], construing Article II, Section 2.
6
Q
- The navy wanted to build a naval base on the north shore of an island in Hawaii. Situated along the north shore of this island were coral reefs which are the home of a very rare species of fish that are found only along the north shore area.Congress conducted hearings to decide whether to authorize the construction of the naval base. During the hearings, one of the speakers who addressed the congressional committee was a famous expert on oceanography and marine biology. The expert vehemently opposed the naval plan and stated that the construction would, in his opinion, result in the extinction of the rare species of fish. Congress thereafter approved the construction of the naval base and passed a bill providing necessary authorization and funding for the project.The expert has filed an action in federal district court seeking to enjoin the construction of the naval base on ecological grounds. Does the expert have adequate standing?(A) Yes, because he has a personal stake in the litigation.(B) Yes, because he is a recognized expert on marine biology and he testified at the congressional hearings.(C) No, because the suit presents a nonjusticiable political question.(D) No, because he is not suffering any actual harm or injury.
A
- (D) The Supreme Court has said that the core of Article Ill’s requirement for cases and controversies is found in the rule that standing is limited to those who allege that they personally have suffered or imminently will suffer an injury. In City of Los Angeles v. Lyons, 461 U.S. 95 (1983), the Court stated that “the plaintiff must show that he has sustained or is immediately in danger of sustaining some direct iniury as a result of the challenged action.” With respect to standing in environmental cases, the Supreme Court has handed down two conflicting rulings. In Sierra Club v. Morton, 405 U.S. 727 (1972), the Sierra Club sought to prevent construction of a ski resort in Mineral Valley in California. The Court ruled that the Sierra Club lacked standing because there was no allegation that any of the Sierra Club’s members ever had used Mineral Valley. By contrast, in United States v. Students Challenging RegulatoryAgency Procedures, 412 U.S. 669 (1973), the Court upheld the standing of a group of George Washington law students alleging that an administrative rate hike would decrease recycling, which would lead to more use of natural resources and increased pollution. The students maintained that their enjoyment of the forests, streams, and mountains in the Washington, D.C., area would be lessened as a result. The Court upheld the group’s standing because the plaintiffs actually used the environmental area for hikes, biking, and other recreational purposes. In this hypo, there is no showing that the expert is making any personal use of the coral reef area. Therefore, he would lack standing on environmental grounds. Choice (A) is incorrect. The Supreme Court has said that the core of Article Ill’s requirement for cases and controversies is found in the rule that standing is limited to those who allege that they personally have suffered or imminently will suffer an injury [City of LosAngeles v. Lyons, 461 U.S. 95 (1983)]. Professor has not suffered nor will suffer some personal injury. His grievance can be shared by anyone who is interested in preventing damage to the environment. Answer (B) is incorrect. This would seem an intelligent answer, but the expert still lacks standing because standing is limited to those who allege that they personally have suffered or imminently will suffer on injury [City of Los Angeles v. Lyons, 461 U.S. 95 (1983)]. The expert’s expertise cannot compensate for his lack of personal injury. Choice (C) is not correct. The political question doctrine would not permit the court to consider the case because of one or more of the following reasons. One, if the U.S. has appropriated the decision- making on this issue to another branch of the federal government. For example, the judiciary cannot hear cases interpreting the rules for impeachment; only the Senate may do so [U.S. v. Nixon, 418 U.S. 683 (1974)]. Two, if the court feels that, for practical reasons, it should not hear the case. For example, the Supreme Court felt it impractical to supervise electoral districts to prevent partisan gerrymandering. [ Vieth vjubelirer, 541 U.S. 267 (2004)].The extinction of the rare species of fish may present serious environmental problems but does not present issues that are related to the political question doctrine.
7
Q
- There are 2,000 insurance agents licensed under the general licensing laws of a state. The state legislature recently passed a highly controversial bill that, inter alia, reduces “good driver” automobile insurance rates and prohibits price-fixing and discrimination by insurance companies. This bill passed despite a well-financed and intense lobbying effort mounted by the insurance industry. After this law was enacted, an insurance agent who is a resident of the state, was interviewed by the local newspaper and publicly stated, “the legislature is a bunch of self-serving crooks.” After the statement made newspaper headlines, the state legislature enacted a statute providing that “the state insurance license of the insurance agent is henceforth revoked.” Astonished by the legislature’s action, the insurance agent, who remains unrepentant, seeks your legal advice.You should advise the insurance agent that the state statute revoking her state insurance license is(A) constitutional, because a state license is a privilege and not a right and, therefore, it is not protected by the due process clause.(B) unconstitutional, because it denies her the equal protection of the laws.(C) unconstitutional, because it is a prohibited bill of attainder.(D) unconstitutional, because it is a denial of a privilege or immunity protected by Article IV.
A
- (C) A bill of attainder is any form of legislative punishment of a named group or individual without judicial trial. Under Article I, Sections 9 and 10, federal and state governments are prohibited from enacting bills of attainder. The statute revoking the state insurance license of the insurance agent is a sufficient form of legislative “punishment” to constitute an impermissible bill of attainder. Choice (A) is incorrect. A state-issued license can be a right protected under the due process clause. Here, the insurance agent did not receive fair notice and a hearing, normal requirements of due process. Choice (B) is incorrect. The equal protection clause provides heightened protection for groups whose legal classifications are suspect (e.g., classifications based on race, ethnicity, nationality, and religion) and for groups whose legal classifications are quasi suspect (e.g., gender, illegitimacy). The insurance agent may have been the subject of discrimination by the legislature but not because she is a member of a suspect or quasi-suspect class. Answer (D) is not the best choice. The privileges and immunities clause of Article IV would prevent the state legislature from discriminating against those who are out-of-staters. Even if the legislature discriminated against the insurance agent by revoking her license, it did not discriminate against her because she is an out-of-stater.
8
Q
- Evidence at Congressional hearings has indicated that thousands of elderly citizens die each year because they cannot afford proper medical and hospital treatment. Based on the evidence presented at these hearings, a senator wishes to introduce a bill providing free medical and hospital care for all citizens 70 years of age and older. The senator, however, intends to include a provision in the bill denying such medical and hospital care benefits to aliens (in the same age category) unless they have been legally admitted for permanent U.S. residency. The senator has asked for your advice regarding his proposed bill. You are understandably concerned about the legality of the aliens proviso.Which of the following is the strongest argument in support of the constitutionality of the provision disqualifying aliens from receiving such medical and hospital benefits?(A) Due to the fact that the medical and hospital benefits will be paid for by the government, they are a privilege not a right and, therefore, are not within the meaning of the Fifth Amendment.(B) The disqualifying provision does not unduly burden either interstate commerce or the right of aliens to travel freely from state to state.(C) The principles of equal protection apply against the states and not against the federal government.(D) The disqualifying provision is reasonably related to legitimate congressional objectives under its immigration, citizenship, and spending powers.
A
- (D) In light of the decision in Mathews v. Diaz, 426 u.s. 67 (1976), choice (D) is the best answer. In Mathews, the Court applied “relaxed scrutiny” in upholding the validity of a federal statute that conditioned an alien’s eligibility for participation in a federal medical insurance program on the satisfaction of a durational residency requirement, but imposed no similar burden on citizens. The Court was at pains to emphasize that Congress, as an aspect of its broad power over immigration and naturalization, enjoys rights to distinguish among aliens that are not shared by the states. Although (C) is a correct statement of law, it does not provide any rationale supporting or justifying the federal government’s treatment of aliens. Choice (A) is incorrect. That the government is paying for something does not necessarily turn something from a right (which is presumably protected) to a privilege (which presumably isn’t). We still have “state action” and thus the government is bound to respect the Constitution even as it doles out supposed privileges. Choice (B) is incorrect. This seems to be a plausible answer because the dormant commerce clause only governs states and local governments; here, we have Congress. But the fact that Congress does not violate the dormant commerce clause is not the best answer because the call of the question focuses on aliens and, hence, on issues of equal protection. Choice (C) is incorrect. It’s true that the 5th Amendment’s Due Process Clause does not contain an equal protection clause. But the Supreme Court has read into it a doctrine of equal protection that is analogous to the 14th Amendment’s Equal Protection Clause [Boiling v. Sharpe, 347 u.s. 497 (1954)].
9
Q
- A city has undergone a massive redevelopment project aimed at remodeling and beautifying the downtown area. Recently the city council passed an ordinance prohibiting the placement of any sign with dimensions larger than six feet on the exterior of any commercial building. Furthermore, signs within the guidelines of the ordinance could only relate to advertising the business of the property’s occupant. The intended purpose of the ordinance was to advance the municipality’s interests in traffic safety and aesthetics.An owner of an office building in the newly developed section of town, placed a 10-foot sign on the outside of his building endorsing his brother’s political candidacy. The city council has ordered the owner to remove the sign from his building, but he has refused to take it down until after the election.If the owner challenges the city ordinance, the most likely result is that he will(A) prevail, because the ordinance violates his freedom of speech.(B) prevail, because such a time, place, and manner restriction on private property is discriminatory and overbroad.(C) not prevail, because although commercial speech is protected by the First Amendment, it is subject to greater regulation than other forms of protected speech.(D) not prevail, because the ordinance is rationally related to a legitimate state interest.
A
- (B) In Metromedia, Inc. v. San Diego, 453 u.s. 490 (1981), the Court struck down a San Diego ordinance which, inter alia, prohibited all billboards containing non-corn mercial messages, except for those messages falling within certain defined categories (e.g., temporary politicaL campaign signs, signs carrying news items or telling the time or temperature, etc.). The Court objected to this handling of non-commercial speech on the grounds that “the city may not choose the appropriate subjects for public discourse”; the city was required to either allow all, or no, non-commercial messages. Here, the Middletown ordinance would have been valid if it were limited to restricting the size of billboards. But since the ordinance also attempted to impose non-content-neutral regulations (by requiring that signs relate only to advertising the business of the owner-occupant), it probably will be struck down under the strict (and often fatal) scrutiny test. Choice (A) is not the best choice. This is an alluring answer, but it’s not the best choice. We want to see what, if anything, is defective with the statute that regulates the speech. So Choice (B) is a stronger choice. Choice (C) is incorrect. It’s true that commercial speech is subject to greater regulation than, say, political speech. But the ordinance in the example indulges in content regulation. So the ordinance will be subject to strict scrutiny and most likely will fail such scrutiny. Choice (D) is incorrect. Strict scrutiny review applies to speech—except for unprotected speech, which is governed by the rational basis test, and time, place, and manner restrictions.
10
Q
- For many years, persons engaged in real estate transactions in a state have utilized installment land contracts. The so-called installment land contract has been popular as a substitute for the mortgage or deed of trust. Under such a contract, the seller agrees to accept, and the buyer agrees to pay, the purchase price in installments over a stipulated period of time. The vendor retains legal title as security for the payment of the purchase price; and the vendee has equitable title under the doctrine of equitable conversion. The most important characteristic of the installment land contract, however, is the forfeiture clause, which provides that if the vendee is in default for 30 days, the vendor may, at his option, declare a forfeiture of the contract, in which case all payments made shall be forfeited as “liquidated” damages and the buyer shall become a tenant at will of the seller.Over the years, many sellers of property under such installment land contracts have declared forfeitures in situations where the prospective buyers were delinquent in their payments, even when the buyer was late with a single payment after nearly all the other payments had been made. In order to remedy this inequitable situation, the state legislature enacted a statute requiring any seller attempting to declare a forfeiture of an installment land contract to do so by instituting a formal foreclosure action in the courts. The new law also provided that prior to the commencement of such a foreclosure action, the seller under such an arrangement must give the buyer a 60-day grace period to make up any delinquent installment payment with interest. The new law expressly applied both to installment land contracts entered into subsequent to its enactment and to installment land contracts already in existence at the time of its enactment.Is this new state statute likely to be held constitutional?(A) Yes, because it is a reasonable regulation of the procedure to be followed in such cases and does not substantially diminish the underlying obligations of the buyer.(B) Yes, because the authority to enact laws regulating real estate sales transactions occurring within the boundaries of individual states is reserved exclusively to the states by the Tenth Amendment.(C) No, because application of the law to installment land contracts entered into prior to its enactment is a violation of the obligation of contracts.(D) No, because application of the law to installment land contracts entered into before or after its enactment is a deprivation of a proprietary interest without due process of the law.
A
- (A) Although nowhere mentioned in the Constitution, the concept of police powers in the 10th Amendment has developed closely with that of the dormant commerce power to enable a state to regulate issues relating to health, safety, morals, and welfare, including economic welfare. As a general rule, “where the state regulates even-handedly to effectuate a legitimate local public interest, and its effects on interstate commerce are only incidental, it will be upheld unless the burden imposed on such commerce is clearly excessive in relation to the putative local benefits” [Pike v. Bruce Church, Inc. 397 U.s. 137 (1970)]. The state law requiring sellers under “installment land contracts” to institute foreclosure actions to relieve the harsh effect of forfeiture by allowing the buyer a 60-day reinstatement period serves a legitimate purpose and leaves unchanged the initial monetary obligations of both parties. Therefore, it will be upheld. Choice (A) is correct. In order to protect the health, safety, morals, and welfare of its citizens, a state may enact legislation that impairs contracts under certain conditions. The action of the Utah legislature may be viewed reasonably as a permissible police power modification of the “installment land contract” because no prior underlying contractual obligation has been impaired, since the buyer must still pay in full the original bargained-for purchase price. In general, private parties who enter into contracts may not stop the legislature from exercising its proper police power for the public good. Nowak, p. 376. Choice (B) is incorrect because the authority to enact laws regulating intrastate real estate sales transactions is not reserved exclusively to the states under the 10th Amendment. Such transactions have a cumulative impact on interstate commerce and are subject to the federal commerce power. The court has refused to use the 10th Amendment to reserve subject areas of commerce for state regulation. Choice (C) is also incorrect. Remember that there is no fundamental right to contract in the Constitution. The state’s regulation thus will be subject to rational review. Under rational review, the court probably will find that there is a legitimate government interest in the state’s desire to protect buyers against unfair contracts. Next, the court probably will find that there are rationally related means. In fact, for the right of contract, the Supreme Court has gone so far as to surmise potential justifications under rational review forthe state. [Williamson v. Lee Optical, 348 U.S. 483 (1955)]. Choice (D) is incorrect. Subsequent to the state’s legislation, the bank still retains its right to foreclose on the property. Accordingly, there is most likely no deprivation of property under the due process clause.
11
Q
- Congress enacted a statute aimed at “countries assisting or furnishing aid or support to nations or movements engaged in hostilities with the United States.” Section 5 of that act authorized and directed the Treasury Department “to issue orders barring entry into the United States any category of goods the sale of which in the United States is likely to improve the economic or balance of payments posture of an assisting country.” The Secretary of State was authorized by Section 6 to define “assisting countries.” Pursuant to Section 5, the Treasury Department issued a regulation, which provided in part that:“Imports of the following categories of goods from assisting countries are hereby prohibited:• . (c) Bulk shipments for resale within the United States of books, pamphlets, flags, decorations or other symbols, excepting, however, scientific, technical, and literary works intended for scholarly purpose…”The State Department designated a foreign country, as an “assisting country,” on the basis of its determination that medical supplies collected by public donation in this country had been sent to another country currently engaged in hostilities with the United States. As a consequence, the Treasury Department issued an order barring practically all products and goods from this foreign country into the United States.A distributor of state and foreign flags, has had a lucrative contract with the foreign country for the importation and sale of their flags in the United States. However, because of the Treasury Department’s order, the foreign country is now barred from importing any of its flags into the United States.In an appropriate federal court, the distributor brings a suit against the Secretary of State and the Treasury Department to set aside the order barring the foreign country’s imports on the grounds that it is inconsistent with the principles of our constitutional form of government. Which of the following is the most proper disposition of the distributor’s suit by the federal court?(A) Suit dismissed, because the distributor does not have standing to bring this action.(B) Suit dismissed, because there is no adversity between the distributor and the defendants.(C) Suit dismissed, because it presents a nonjusticiable political question.(D) Suit dismissed, because it presents a moot issue.
A
- (C) The doctrine of political questions precludes federal judicial review of nonjusticiable issues (i.e., matters not capable of judicial resolution due to inherent separation of powers limitations). The leading case of Baker v. Carr, 369 U.S. 186 (1962), established a test to identify such nonjusticiable issues. For example, where a textually demonstrable commitment to a coordinate political branch of government has been made, or where there is a lack of judicially manageable standards to resolve the issue, or where judicial resolution would create the potential for embarrassment from multifarious pronouncements by various branches of government, then the judiciary will decline resolution of such nonjusticiabie political questions. Choice (C) is correct because foreign affairs is a nonjusticiable political question over which federal courts will not exercise jurisdiction. Choice (A) is incorrect. Actually, the distributor does have standing insofar as it can show that: The distributor has suffered a direct harm from the governmental law; the governmental law is a “but for” cause of the distributor’s harm; and there is a “substantial likelihood” that the relief sought from the court by the distributor, if granted, would remedy the harm. Choice (B) is incorrect. In order to sue, the distributor would have to show that it has suffered a material harm under the adversity requirement. The facts tell us that the distributor has a “lucrative” contract with the foreign country. Hence, we may assume that the governmental law, by prohibiting this contract, would satisfy the adversity requirement. Choice (D) is incorrect. The case will not be dismissed for mootness because an actual controversy does exist, however the court will refrain from hearing that controversy because it is a nonjusticiable political question.
12
Q
- A state enacted a statute prohibiting any motor vehicle traveling within the state from having window tinting or glass coating of any kind. The bill passed the state legislature at the urging of state and local law enforcement agencies who argued that tinted windows prevented them from observing interior car activity. Most citizens also supported the bill, especially after a state trooper was killed by an occupant in a window-tinted limousine. The trooper was unable to see that his assailant was armed and dangerous when he approached the vehicle for a speeding infraction.A limousine company operates a limo service in a neighboring state and has a fleet of 68 limos, all of which have tinted windows. Each year, the company makes thousands of trips into the state to transport passengers to the international airport situated 10 miles from the border separating the two states. Since all of its limos are manufactured with tinted windows, the company will incur great expense in ordering limos without tinting.The company brings suit to challenge the constitutionality of the state statute. Assuming that the company has proper standing to assert such an action, which of the following is their strongest constitutional argument to invalidate the aforesaid statute?(A) Because window tinting is permitted on vehicles in neighboring states, this law denies the company the equal protection of laws.(B) Because this law burdens interstate commerce by prohibiting all vehicles with window tinting from entering the state, this law violates the commerce clause.(C) Because window tinting on vehicles is legal in their home state, this law violates the contract clause by preventing the company from fulfilling its obligation to transport passengers into a neighboring state.(D) Because interstate travel is a fundamental right that may not be burdened by state law, it violates the company’s substantive due process rights by arbitrarily and unreasonably regulating economic activity.
A
- (B) Under Article I, Section 8, Congress shall have the power “To regulate Commerce with foreign Nations, and among the several States, and with the Indian Tribes.” This plenary federal commerce power is held concurrently with the states, which may freely govern matters that do not require uniform national regulation. For a state regulation affecting interstate commerce to pass judicial scrutiny, the statute must be found reasonable and nondiscriminatoryupon balancing the benefit to the state against the burden imposed on interstate commerce such that no less-restrictive alternative means of regulation is available. Where a state statute imposes a discriminatory burden on out-of-state interests, its validity is seriously in question. The statute effectively prevents the company from doing business in its state. The company’s strongest constitutional argument to invalidate the statute would be the Commerce Clause challenge presented in choice (B). Choice (A) is incorrect because all motorvehicles in the state are treated equally under the law. Choice (D) is incorrect because strong support is given in the facts to demonstrate that the purpose for enacting the statute was neither arbitrary nor unreasonable. Finally, choice (C) is incorrect because the contracts clause of Article I, Section 10 states that no state shall impair the obligation of contracts. Here, no state has prevented the company from fulfilling its contractual obligations, although it may now have to pay more to do so.
13
Q
- The state registrar of elections refused to put a candidate’s name on the ballot because the registrar believed that the candidate was not a resident of the state. The registrar contended that Article I, Section 2 of the U.S. Constitution specifically required a candidate for the House of Representatives “be an Inhabitant of that State in which he shall be chosen.” As a consequence, the candidate filed suit in state court against the registrar of elections seeking to have her name placed on the ballot. The state court ruled in her favor and determined that the candidate did, in fact, qualify as a resident of the state. The registrar appealed the decision to the state supreme court which, in turn, affirmed the lower court ruling. Thereafter, the candidate’s name was placed on the ballot. After a hotly contested election, the candidate narrowly defeated the incumbent.However, at the time that the candidate was to be seated as a House member, the House of Representatives held hearings on her qualifications and eligibility. By a two-thirds vote, the House determined that the candidate was not a resident of the state at the time of her election and refused to seat her. The candidate then brought suit in federal district court against the Speaker and other officers of the House, seeking back pay and an order that she be seated in the House of Representatives. The defendants demurred, claiming lack of subject matter jurisdiction.Which of the following is the strongest constitutional argument supporting the defendants’ demurrer?(A) There is no case or controversy between the candidate and the officers of the House of Representatives.(B) The case presents a nonjusticiable political question.(C) The suit should have been brought as an original action in the U.S. Supreme Court.(D) Under Article III of the Constitution, the federal courts are not empowered to render advisory opinions.
A
- (B) This Constitutional Law question focuses on the specific case of Powell v. McCormack, 395 U.S. 486 (1969), but there is an important twist. In Powell, the House of Representatives refused to seat him (after he had been elected) due to the fact that he had wrongfully diverted House funds for his own personal use. In brief, the Court held that the House did not have authority to exclude him since he met all the requirements for membership expressly prescribed in Article I, Section 2 (namely, age, citizenship, and residency). Thus, the Court held that Powell’s claim did not present a political question. In this example, however the issue is whether Kennedy fulfilled the “residency” requirements set forth in Article I, Section 2. Since the House has the unreviewablepowerto determine the “residency” (as well as “age” and “citizenship”) qualifications for its members, the case presents a nonjusticiable political question. Choice (A) is incorrect. Because Powell had demanded back pay (as the candidate had done), the Court held that “a prayer for specific relief prevented a finding of mootness and demonstrated that there was, in fact, a ‘case or controversy.” See Powell, 395 U.S. at 497. Choice (C) is incorrect. Artide Ill, Section 2 states: “In all cases affecting ambassadors, other public ministers and consuls, and those in which a state shall be party, the Supreme Court shall have original jurisdiction. In all the other cases before mentioned, the Supreme Court shall have appellate jurisdiction, both as to law and fact, with such exceptions, and under such regulations as the Congress shall make.” The candidate’s lawsuit does involve governmental officials but not those who are covered by the Supreme Court’s original jurisdiction. Choice (D) is not the best choice. While this summation of Article Ill is correct, the federal court is not being invited to issue an advisory opinion. The court is being invited to adjudicate an actual controversy.
14
Q
- There has been a great deal of news media coverage regarding the problem of underage drinking. Many high school and college students have been purchasing fraudulent driver’s licenses with phony birthdates showing “proof’ of being 21 years of age or older. As a consequence, many teenagers are able to purchase alcoholic beverages at liquor stores, restaurants, and bars. The situation is becoming especially alarming on many college campuses. A magazine article reported that the majority of freshmen between the ages of 18 and 19 at many colleges had illegally purchased evidence of phony “proof.”With underage drinking reaching epidemic proportions, Congress enacted a statute establishing a federal commission to monitor and curtail alcoholic beverage sales to underage drinkers. To implement the program on a national scale, the bill required each state to pass legislation establishing a local “watchdog” agency to facilitate compliance with congressional intent.A state has filed suit challenging the constitutionality of the federal statute. The law is likely to be held(A) valid, because the sale of alcoholic beverages has a substantial impact on interstate commerce.(B) valid, because the establishment of a state watchdog agency under the auspices of a federal regulatory scheme is consistent with the provisions of the Twenty First Amendment.(C) invalid, because it violates the fundamental principles of state sovereignty embodied by the Eleventh Amendment.(D) invalid, because the federal government may not compel state legislatures to enact and enforce a federal regulatory program.
A
- (D) The landmark case of New York v. United States, 505 U.S. 144 (1992) clearly limited Congress’s right to interfere with a state’s ability to make and apply laws through legislative, judicial, and administrative functions. Based on principles of state sovereignty under the 10th Amendment, Congress may not simply commandeer the legislative processes of the States by directly compelling them to enact and enforce a federal regulatory program. The Court explained that allowing Congress to commandeer state governments would undermine government accountability because Congress could make a decision but the states would take the political heat and be held financially and legally responsible for a decision that was not theirs. Therefore, if a federal law compels state legislative or regulatory activity, the statute is unconstitutional even if there is a compelling need for the federal action. Chemerinsky, p. 233. Under its commerce power, Congress could clearly pass legislation to establish a federal commission to monitor and curtail alcoholic beverage sales to underage drinkers. However, a 10th Amendment violation arises when implementation of the bill requires each state to pass legislation to establish a local “watchdog” agency to facilitate compliance with congressional intent. Congress cannot compel state regulatory activity. The federal statute will be held invalid under the 10th Amendment. Therefore, choice (D) is the correct answer. Choice (A) is not the best answer. It is true that the sale of alcoholic beverages has a substantial impact on interstate commerce. And the commerce clause of Article I, Section 8 states that Congress may regulate commerce with foreign nations, amongthe states, and with the Indian tribes. Accordingly, it would seem likely that Congress may create a watchdog agency to regulate some aspect of alcohol sales. However, New York v. United States, 505 U.S. 144 (1992) held that Congress’s exercise of its commerce clause powers may not infringe on the states’ rights under the 10th Amendment. Specifically, the Supreme Court explained that Congress may not simply commandeer the legislative processes of the States by directly compelling them to enact and enforce a federal regulatory program. That would be deemed coercive and violate of the 10th Amendment. Choice (B) is incorrect. The 21st Amendment repealed the 18th Amendment, which had prohibited the manufacture, sale, or transportation of intoxicating liquors. Congress probably will not turn to the 21st Amendment to justify its law. Even if Congress did, it would probably violate the states’ 10th Amendment rights against the federal commandeering of state governments. Choice (C) is incorrect. The 11th Amendment states: “The Judicial power of the United States shall not be construed to extend to any suit in law or equity, commenced or prosecuted against one of the United States by Citizens of another State, or by Citizens or Subjects of any Foreign State.” The 11th Amendment thus prevents lawsuits by private citizens against the states; it does not protect states from the federal government.
15
Q
- A women’s action group attempted for many months, unsuccessfully, to reach an agreement with the local professional men’s club to admit women to membership. The women’s group instituted a suit for a declaratory judgment in federal court to determine whether the men’s club was subject to the state’s anti-discrimination act.Prior to the elections for city officials, four members of the women’s group were sent to picket the offices of the mayor and district attorney, both prominent members of the men’s club. Two members walked outside the front of the mayor’s office building, carrying signs that read, “The mayor is supposed to serve all the people but his lunch club is for men ONLY. So don’t vote for him.” The other two pickets walked outside the rear of the district attorney’s office building, carrying similar signs, telling the public not to vote for him. This picketing was carried on from 9 A.M. to 5 P.M.The same day, two more pickets were assigned to carry identical signs in front of the mayor’s official residence. Two pickets also carried duplicate signs in front of the district attorney’s suburban home during the early evening hours. The picketing at all sites was held peacefully without any disturbance. The relevant city ordinances concerning picketing read as follows:“Section 201. No picketing shall be permitted inside of, or on any sidewalk or street immediately adjacent or contiguous to, city hall, without express permission of the mayor. Applications for such permission shall be filed at least three days before such picketing is intended to begin and shall state the purpose, place, and time of the proposed picketing.Section 202. It shall be unlawful for any person to engage in picketing before or about the residence of an individual. Nothing herein shall be deemed to prohibit the holding of a meeting or assembly on any premises used for the discussion of subjects of general public interest.”The federal district court will most likely avoid making a decision on the merits of the suit for declaratory judgment because(A) the case lacks adequate ripeness.(B) there is no case or controversy.(C) the relief sought is essentially for an advisory opinion.(D) the women’s group lacks standing.
A
- (C) When a federal constitutional claim is premised on an unsettled question of state law, the federal court should “stay its hand” (abstain temporarily), so as to give state courts a chance to settle underlying state law questions. In the present case, the women’s group initially should have brought suit in state court (thereby exhausting all state remedies) before seeking federal judicial review. Clearly, the women’s group is presently “harmed” by the men’s club’s exclusionary membership policy. Therefore, choice (A) is incorrect. Choice (B) is wrong since an actual “case and controversy” is presented by the men’s club’s exclusion of women members. Choice (D) is incorrect. The women’s group would seem to possess standing here: The men’s club has inflicted injury against the women’s group’s members by excluding them; the court can offer a remedy for its injury.
16
Q
- A teachers union, a nongovernmental entity, seeks to picket the local city school board for its decision to require higher certification for instructors who wish to teach in the gifted and talented classes in elementary school. After a few days of picketing, the school board seeks a temporary injunction in the state court to restrain further picketing of the school board. The school board insists that the teachers union has violated Section 101 of the city’s picketing ordinance. Section 101 reads as follows:“Section 101. No picketing shall be permitted inside of, or on any sidewalk or street immediately adjacent or contiguous to public elementary and secondary schools without express permission of the mayor. Applications for such permission shall be filed at least three days before such picketing is intended to begin and shall state the purpose, place, and time of the proposed picketing.”The court will most likely(A) grant relief, because the teachers unionfailed to follow the procedure outlined in theordinance.(B) grant relief, because the ordinance isconstitutional.(C) deny relief, because the picketing ordinancewas unconstitutional on its face.(D) deny relief, because the ordinance does notprovide procedural due process.
A
- (C) A frequently tested Constitutional Law area deaLs with Licensing statutes. As ageneral rule, whenever an ordinance is “void on its face,” the defendant need notapply for a permit. In this situation, he is entitled to deliver his speech and can successfuLly defend any subsequent prosecution. An ordinance may be deemed to befacially invalid for the following three reasons: (1) “vagueness,” (2) “overbreadth,”(3) “unfettered discretion” in the licensing official. On the other hand, when an ordinance is valid on its face (although a permit may be arbitrarily denied), the applicant must seek judicial relief before engaging in his speech. See Poulos v. New Hampshire, 345 U.S. 395 (1953). Choice (A) is incorrect. This would be true if the ordinance were constitutional. Here, the ordinance is invalid on its face because it affords the mayor unfettered discretion to deny permits for picketing. Under the ordinance, the mayor may deny permits to those advocating viewpoints that he finds objectionable. Choice (B) is incorrect. The ordinance is unconstitutional because it affords the mayor unfettered discretion to deny permits for picketing. Under the ordinance, the mayor may deny permits to those advocating viewpoints that he finds objectionable. Choice (D) is not the strongest answer. This is a potentially good answer. However, the ordinance will be struck down on its face because it affords the mayor unfettered discretion to deny permits for picketing. Under the ordinance, the mayor may deny permits to those advocating viewpoints that he finds objectionable.
17
Q
- A group of students from a public state college decide to picket the college for raising tuition. After three days of picketing, the college asks a court to grant a temporary injunction against the picketers. The college claims that the picketers have violated Section 201 of the city’s picketing ordinance. Assume that the court granted the temporary injunction against the students for picketing the college.“Section 201. No picketing shall be permitted inside of, or on any sidewalk or street immediately adjacent or contiguous to public colleges without express permission of the mayor. Applications for such permission shall be filed at least three days before such picketing is intended to begin and shall state the purpose, place, and time of the proposed picketing.”In a subsequent action challenging the constitutionality of Section 201 of the city’s picketing ordinance, the court will most likely rule that the section is(A) constitutional, because the ordinance is a valid exercise of the state’s police power.(B) constitutional, because the ordinance is within the reserved rights of the states under the Tenth Amendment.(C) unconstitutional, because the ordinance is void for vagueness and overbreadth.(D) unconstitutional, because the ordinance violates petitioner’s rights under the First Amendment.
A
- (D) In Coxv. Louisiana, 379 U.S. 537 (1965), the U.S. Supreme Court held that an analogous ordinance prohibiting “parades or demonstrations along any street except in accordance with a permit issued by the police chief’ was clearly unconstitutional. The Court reasoned that it would be an arbitrary exercise of state police power to enable a public official to determine which expressions of view will be permitted and which will not. Allowing unfettered discretion in local officials in the regulation of the use of streets is an unwarranted abridgement of one’s freedom of speech and assembly as secured by the 1st Amendment and applied to the States by the 14th Amendment. Choice (C) is not the best answer because the statute is neither over- broad nor vague; it is invalid because it gives the mayor the “unfettered discretion” to approve or disapprove permit applications. Choice (A) is incorrect. Courts have stated that the 10th Amendment contains “police powers,” which beLong to local governments. Those police powers permit local governments to regulate speech for purposes of health, safety, welfare, and morals. However, local governments may not pass laws that regulate speech and which are invalid on their face. We have such a law here because it gives the government unfettered discretion to discriminate against certain viewpoints. Such a law is facially invalid. Finally, choice (B) is incorrect. The state is said to have “police powers” under the 10th Amendment, rather than “reserved powers.” Regardless of the term, a state may not use its 10th Amendment powers to pass a law that can permit the state to enjoy unfettered discretion to discriminate against certain viewpoints. Such a law is facially invalid.
18
Q
- A city passed a picketing ordinance. Section 202 of that ordinance reads:“Section 202. It shall be unlawful for any person to engage in picketing before or about the residence of an individual. Nothing herein shall be deemed to prohibit the holding of a meeting or assembly on any premises used for the discussion of subjects of general public interest.”A group of teachers who work for a public school inside of the city were angry that the principal decreased their salaries. The teachers reacted by picketing on the lawn of the principal’s privately owned house. The teachers were charged with violating Section 202 of the ordinance. The teachers have filed suit attacking the constitutionality of Section 202.The court will most likely declare Section 202(A) constitutional, as a permissible time, place, and manner regulation.(B) constitutional, as a valid exercise of a state’s reserved powers under the Tenth Amendment.(C) unconstitutional, as discriminatory on its face. (D) unconstitutional, as vague and overbroad.
A
- (A) The government may place reasonable restraints on the time, place, and manner of speech in public areas, such as streets, sidewalks, and parks. A public street does not lose its status as a traditional public forum simply because it runs through a residential neighborhood. A three-part test is used to determine the constitutionality of time, place, or manner regulations of speech and assembly in public places. The regulation must: (1) be content-neutral; (2) be narrowlytailored to serve a significant government interest; and (3) leave alternative channels of communication open. Section 202 is content-neutral (the regulation does not prefer some messages over others). It serves the significant government interest of protecting residential privacy. An important aspect of such privacy is the protection of unwilling listeners within their homes from the intrusion of objectionable or unwanted speech. Moreover, Section 202 is narrowly tailored to serve that governmental interest, since, although its ban is complete, it targets and eliminates no more than the exact source of the “evil” it seeks to remedy: offensive and disturbing picketing focused on a “captive” home audience. It does not prohibit more generally directed means of public communication that may not be completely banned in residential areas [Frisby v. Schultz, 487 U.S. 474 (1988)]. Therefore, choice (A) is correct as it comes to the right conclusion and highlights the correct legal issue. Choice (B) is a potentially good answer but certainly it is less precise than (A). Choice (C) is incorrect. For a law that regulates free speech to be declared invalid on its face, it must suffer from one or more of these: (1) “vagueness,” (2) “overbreadth,” (3) “unfettered discretion” in the licensing official. Section 202 does not suffer from any of these. Choice (D) is incorrect. Section 202 is not so vague or overbroad as to be unconstitutional. Alaw is deemed unduly vague if a reasonable person is unable to distinguish what is permitted [City of Chicago v. Morales, 527 U.S. 41 (1999)]. Here, the law is not unduly vague because a reasonable person should be able to determine when picketing is taking place before or about the residence of an individual. A law suffers from overbreadth when it punishes speech that is otherwise protected [Schad v. Borough of Mt. Emphraim, 452 U.S. 61 (1981)]. Here, a person has limited constitutional right of free speech before or about a person’s private property, especially that of a home.
19
Q
- A state imposes a graduated income tax upon net income calculated under federal law. Section 22 of the city’s reform act of 2010, which is to become effective on January 1, 2010, provides: “Any parent or guardian financially responsible for the education of his ward may claim a direct tax credit against his income tax liability equal to the amount of tuition of a child or children of high school age who does not attend a public high school.” Other provisions define “tuition” very broadly but limit the credit to tuition paid to schools meeting the educational requirements as determined by the state.On December 1, 2009, the parents of two students who attend a public high school in the state sue for a declaratory judgment and injunction in federal court, claiming that Section 22 violates the establishment clause of the First Amendment. The federal court will most likely(A) dismiss the action, because the suit involves a political question.(B) dismiss the action, because the plaintiffs lack standing as taxpayers.(C) dismiss the action, because the issues are not ripe.(D) dismiss the action, because the issues being litigated are moot.
A
- (C) Since the reform act will not become effective until January 1, 2010, the court will dismiss this action for lack of ripeness. It is crucial that a person asking the court to hold a statute unconstitutional be able to show, not only that the statute is invalid, but also that “he has sustained or is immediately in danger of sustaining some direct injury” as a result of its enforcement. Here, the parents must wait until after the reform act has gone into effect before initiating suit. Choice (A) is incorrect. The court cannot hear the case because the cause of action lacks ripeness. Had the case been ripe, the court would not have been precluded from hearing the case because there is no political question at issue on these facts. A political question is defined as an issue that has been committed to another branch of government. Choice (B) is incorrect because taxpayers generally do not have standing to sue as such [Lujan v. Defenders of Wildlife, 504 u.s. 555 (1992)]. There is a reference to taxes, but the aggrieved parents are not suing as taxpayers. The aggrieved parents are instead arguing that they will suffer a specific harm because they send their children to public schools and thus cannot take advantage of the tax credit. Choice (D) is not the best choice. Article Ill states that the jurisdiction of federal courts only extends to “cases or controversies.” That means federal courts generally may not hear cases that are moot, in other words, those cases where the issues being litigated no longer affect the litigants. For example, there is a probLem of mootness in a suit to overturn a criminal conviction after the convicted dies. In this case, the issues are not moot since the aggrieved parents might suffer harm from the statute.
20
Q
- A state passed a statute stating that it will pay for one-half of the tuition for any student attending a private school, whether religious or secular, as long as the school meets the educational requirements established by the department of education.The parents of two students who attend a public high school in the state sue for a declaratory judgment and injunction in federal court, claiming that the state statute violates the establishment clause of the First Amendment. Assume that the federal court decides to hear the case on its merits.Which of the following is the state’s LEAST persuasive argument for sustaining the validity of the statute?(A) The state statute benefits the parents or guardians of all the children in private schools, religious and nonreligious.(B) The primary effect of the state statute is not to advance or inhibit religion.(C) The administration of the state statute by the department of education does not foster excessive governmental entanglement with religion.(D) The state statute is a valid exercise of state regulatory action in the field of education.
A
- (D) Choice (D) is the least persuasive argument because it does not address the establishment clause issue clearly raised by the statute, which forms the basis for the parents’ claims. Even if the statute were a valid exercise of state regulatory action in the field of education, the issue of a potential establishment clause violation would remain. Choices (A), (B), and (C) are correct applications of the law and address the claim raised by the parents. Choice (A) would tend to support the view that the state statute does not violate the establishment clause, since both religionists and nonreligionists can benefit. Choice (B) is a plausible argument insofar as both religionists and non-religionists can benefit from the state statute. Finally, choice (C) is a plausible argument insofar as the government is not required to make determinations about what is a religion.
21
Q
- A state enacts a statute that will allocate funds generated by state taxes to pay for one-half of the annual salaries of teachers in private schools, whether the schools are public or private, as long as the schools satisfy the requirements established by the United States Department of Education.In an action brought by several state taxpayers, all avowed atheists, to challenge the constitutionality of the statute, the enactment will most likely be declared(A) unconstitutional, as violative of the First Amendment’s establishment clause.(B) unconstitutional, as violative of the First Amendment’s establishment clause as applicable through the Fourteenth Amendment.(C) constitutional, as nonviolative of the First Amendment’s establishment clause.(D) constitutional, as being within the area of compelling state interest.
A
- (B) In Lemon v. Kurtzman, 403 U.S. 602 (1971), the U.S. Supreme Court held unconstitutional a statute that provided for payment of salary supplements to parochial schoolteachers who taught solely secular subjects. The Court concluded that the degree of government surveillance necessary to ensure that the supplements would be restricted to teachers of secular subjects would entangle the government excessively. Choice (A) is not the strongest answer. This is a good answer, but there is one glitch. Remember that the Establishment Clause, on its face, prohibits “Congress” from establishing a religion. The Establishment Clause applies to local governments via the 14th Amendment’s Due Process Clause. Choice (C) is incorrect. Most likely, the statute wilL be deemed to violate the Establishment Clause. The Supreme Court rejected as unconstitutional a statute that provided for payment of salary supplements to parochial schoolteachers who taught solely secular subjects. The Court concluded that the degree of government surveillance necessary to ensure that the supplements would be restricted to teachers of secular subjects would entangle the government excessively [Lemon v. Kurtzman, 403 U.S. 602 (1971)]. Choice CD) is incorrect. It is unlikely that the Court will deem Section 40’s interest to be compelling. The Supreme Court has already rejected as unconstitutional a statute that provided for payment of salary supplements to parochial schoolteachers who taught solely secular subjects. The Court concluded that the degree of government surveillance necessary to ensure that the supplements would be restricted to teachers of secular subjects would entangle the government excessively [Lemon v. Kurtzman, 403 U.S. 602 (1971)].
22
Q
- A man entered into a franchise contract with a company to operate a fast-food restaurant in a city. The company’s national headquarters is located in another state. After the contract was executed, the man leased a store in a shopping center where he planned to open his restaurant. City public officials, however, refused to grant the man the necessary food vendor’s license despite the fact that he could prove that his restaurant complied with all provisions of the municipal licensing ordinance. Section 1287 of the city food vending ordinance provides, in part, that “a food vendor’s license shall be issued to any applicant who properly complies with all of the health requirements of this ordinance.” After the man’s application for a food vendor’s license was rejected, he requested a hearing to establish his qualifications. City officials refused this request and also declined to give any reason for his license denial.Which of the following is the strongest constitutional argument that the man may use to challenge the refusal of the city officials to grant him a food vendor’s license?(A) The city action denies him procedural due process.(B) The city action denies him substantive due process by arbitrarily regulating economic activity.(C) The city action constitutes an undue burden on the potential interstate commerce between the man and his out-of-state franchisor.(D) The city action impairs the obligation of the man’s contract with the franchising company and his rental agreement with the shopping mall.
A
- (A) Procedural due process guarantees a fair decision-making process before the government takes some action directly impairing an individual’s life, liberty, or property interests under the 5th and 14th Amendments. Where such a deprivation occurs that is serious in nature, the procedural safeguards of notice and opportunity to be heard (i.e., hearing) are available. Choice (B) is incorrect. There is no fundamental right of contract or “economic activity” under the “substantive due process” clause of the 14th Amendment [West Coast Hotel v. Parish, 300 U.S. 379 (1937)}. Choice (C) is incorrect because in order to establish that a local government is presenting an undue burden on interstate commerce and, thus, violating the commerce clause of Article I, Section 8, there must be a showing that a local government is requiring out-of-staters to comply with a law whose benefits to the local government are outweighed by its burdens on out-of-staters. Choice (D) is incorrect. Article I, Section 10 contains the “contract clause,” which states that no state shall pass any law impairing the obligation of contracts. To sue under this clause, a party would have to show that the state action affected a preexisting contract. The contract at issue in this question does preexist the city’s denial of the license. However, according to Energy Reserves Group v. Kansas Power & Light, 459 u.S. 400 (1973), there must also be a “substantial impairment” of the obligation of contracts. In this instance, the denial does not “substantially impair” the obligation because the company theoretically can still enforce its terms against the man and seek damages.
23
Q
- Congress has recently enacted a statute requiring all males between the ages of 18 and 30 to take a physical examination each year. The results of the exam are sent to a government data information center for the purpose of keeping information about men who may be drafted into the military.A 25-year-old law school graduate has herpes. He has recently sent resumes to many governmental agencies. Fearful that the information about his herpes condition will become available, he seeks a declaratory judgment that would forbid the government from requiring him to take a physical examination.Which of the following is the best constitutional basis in support of the federal law?(A) The commerce clause.(B) The dormant commerce clause.(C) To raise and support an army and navy.(D) To provide for the general welfare.
A
- (C) Here, choice (C) is the best answer. under Article I, Section 8, Clauses 11 and 12,Congress has wide war power authority. Certainly, the draft and selective servicesystems have been repeatedly upheld as a proper exercise of Congress’s war powers. Based on the decision in United States v. O’Brien, 391 u.s. 367 (1968), individuals may be constitutionally required to register and submit to examinationsreasonably required by the selective service system to facilitate the conscriptionof manpower for military service. Choice (A) is incorrect. The Commerce Clauseof Article I, Section. 8 states that Congress may regulate commerce with foreignnations, among the several states, and with the Indian tribes. There does notappear to be any commercial transactions in our example, however. Choice (B)is incorrect. No congressional statute can be deemed unconstitutional becauseit violates the dormant commerce clause. The concept of the dormant commerceclause pertains only to local governments, as when a Local government passes a lawthat discriminates against out-of-staters or places an undue burden on interstatecommerce. In our example, Congress has passed a law and hence its law cannot bedeemed to violate the dormant commerce clause. Choice (D) is incorrect. Article I,Section 8, reads: “The Congress shall have Power to lay and collect Taxes, Duties,Imposts and Excises, to pay the Debts and provide for the common Defence andgeneral Welfare of the united States.” The Supreme Court has taken the reference to“general welfare” to mean that Congress enjoys great discretion in how it choosesto allocate money for the pubLic [United States v. Butler, 297 U.S. 1 (1936)]. There isno such allocation of money in our example; just a regulation.
24
Q
- The civic auditorium is owned by the city. The auditorium is rented out to various organizations throughout the year. With a seating capacity of 1,500, it is customarily leased for rock concerts, rodeos, sporting events, fashion shows, etc. In January, a rock star held a week-long concert there, drawing a full house each night. Generally, the city leases the auditorium’s facilities for a charge of $2,000 per day.In February, a religious sect applied to rent the auditorium for its annual celebration. However, the city council voted 7 to 3 against permitting the group from using the auditorium. When their rental application was denied, the religious sect threatened to take legal action against the city council. They contended unfair discrimination inasmuch as other religious organizations had leased the auditorium in previous months for their religious gatherings. Amid this controversy, the city council passed an ordinance prohibiting the rental of the auditorium to any religious group. The ordinance was passed in a “closed door” session, which did not permit any debate or hearings on the matter.Is this newly enacted city ordinance likely to be held constitutional?(A) No, because it violates the due process rights of religious groups by not affording them an opportunity for a hearing.(B) No, because the ordinance discriminates against religious groups in violation of the free exercise clause of the First Amendment, as applied to the states via the Fourteenth Amendment’s due process clause.(C) Yes, because the ordinance treats all religious groups equally.(D) Yes, because a city ordinance is not a state action, per Se, and, therefore, is not subject to the limitations of the Fourteenth Amendment.
A
- (B) During the 1940s and 19505, the Supreme Court invalidated a number of laws that restricted religious practices primarily on the basis that they interfered with the free speech protection of the 1st Amendment. These decisions overturned statutes regulating the dissemination of religious views because they interfered with both the freedom of speech and religion. In sum, the Court struck down ‘icensing systems for demonstrations or meetings that gave government officials discretion to deny licenses on the basis of the content of speech, while upholding others that had permit requirements based on nondiscriminatory “time, place, or manner” factors. In these cases, the fact that religious meetings were suppressed under discretionary statutes indicated a violation of free exercise rights, but the statutes were invalid in total as they conflicted with the free speech clause. By analogy, the city ordinance would be unconstitutional because it unfairly burdens the freedom of religion (as well as speech). Here, the city ordinance does not further a “compelling state interest,” as less restrictive means are available. Choice (A) is incorrect. According to the 14th Amendment’s due process clause, no state (or other local government) may deprive a person of her life, liberty, or property without due process of law. Generally, no hearing or advance notice is required for laws that are passed and do not target a discrete group of individ ua Is [Bi-Metallic Investment Co. v. State Board of Equalization, 239 u.s. 441 (191 5)1. Here, the religionists, while a defined group, probably do not constitute a group that is sufficiently discrete as to merit a hearing. Of course, this preclusion does not close other legal channels forthe religionists. Choice (C) is incorrect. This may be true, but the equal treatment does not negate the city council’s violation of the religious groups’ free exercise rights under the 1st Amendment. Note, too, the 14th Amendment’s equal protection clause, which states that no person shall be denied the equal protection of the laws. Religious groups are considered to be a “suspect class” under the equal protection clause and discrimination against them will be subject to strict scrutiny. The city council probably will be unable to show that it possesses a “compelling governmental interest” under strict scrutiny. Choice (D) is incorrect. The 14th Amendment, like the other amendments to the Bill of Rights (the first eight Amendments), onLy protect a person from governmental abuse. This is known as the “state action” requirement, even though “state action” refers to any governmental action—including action by any federal or any local governmental body. The city ordinance would thus qualify as state action.
25
Q
- A state has enacted a statute imposing a tax on the extraction of all platinum in the state. The extraction of other minerals is not taxed by the state. This is true even though there is considerable mining of silver, turquoise, sulfur, and stone within the state. As a result, the largest platinum mining company in the state has filed suit challenging the constitutionality of the state’s platinum tax statute.Which of the following best states the burden of persuasion if the mining company attacks the statute as violating the equal protection of the laws?(A) The state must convince the court that the classification in the statute is rationally related to the advancement of a legitimate state interest.(B) The state must convince the court that the classification in this statute is the least restrictive means by which to advance a compelling state interest.(C) The mining company must convince the court that the classification in this statute is not necessary to advance a compelling state interest.(D) The mining company must convince the court that the classification in this statute is not rationally related to the advancement of a legitimate state interest.
A
- (D) A highly tested area of Constitutional Law deals with the standards of review used by the courts in equal protection challenges and, specifically, what is the burden of persuasion in each situation? In this question, the state has imposed a statute taxing the extraction of platinum. The tax discriminates against the state’s largest platinum mining company, raising an equal protection challenge, since the state is not taxing the extraction of any other mineral. The standard of review that the court will use in this case will be the traditional rational-basis test. The rational-basis test, which is used for classifications relating to non-suspect classes and to non-fundamental rights, includes the following: classifications based on poverty, wealth, age, public housing, and welfare benefits. See Nowak, Constitutional Law pp. 620-623. under the rational- basis test, the burden is on the plaintiff to prove that the challenged measure is not rationally related to a legitimate state interest. Choice (D) is correct. Note that it is extremely difficult for the plaintiff to prevail under the “mere rationality,” or rational- basis test, because any legitimate interest that is reasonably furthered by enactment of the challenged measure will be purportedly valid. Choice (A) is incorrect. We do not have a suspect or “quasi suspect” cLass that is being created by the law. Hence, the law will be subject to rational review, under rational review, the challenger—not the state—bears the burden of showing that the law does not advance a legitimate governmental interest or that the law’s means are not rationally related. Choice (B) is incorrect. If the state’s law had created a suspect class, it would be subject to strict scrutiny. Under strict scrutiny, one of the things that the state would have to show is that the means were least restrictive. Unlike race, ethnicity, nationality, or religion, the mining company is not a member of a suspect class. Thus, no strict scrutiny is warranted; rational review will be applied. Choice (C) is incorrect. If the state’s law had created a suspect class, it would be subject to strict scrutiny. Under strict scrutiny, one of the things that the state would have to show is that the government’s interests were compelling. Unlike race, ethnicity, nationality, or religion, the mining company is not a member of a suspect class. Thus, no strict scrutiny is warranted; rational review will be applied.
26
Q
- A woman attended a private university. One afternoon she was approached by a photographer for a magazine, who asked her if she would be interested in posing nude for an upcoming issue. The woman, who was an aspiring model, agreed and posed for a number of nude pictures, which subsequently appeared in the magazine.Afterward, administrators and professors at the university began to harass her for what they considered to be imprudent behavior. During class, her instructors frequently called her “Playmate” and “Stripper.” Consequently, the woman brought suit in federal court against the university, alleging harassment, and seeking an injunction and damages. After this action was instituted, the university signed a stipulation agreeing not to harass her in the future.The federal court should now(A) hear the case.(B) dismiss the action as moot.(C) dismiss the action since the issues are no longer ripe.(D) dismiss the action because there is no case or controversy.
A
- (A) The doctrine of standing requires that a person asserting the violation of a constitutional (or statutory) right must show a direct and immediate personal injury due to the challenged action. Choice (B) is incorrect since it is irrelevant whether the university signed a stipulation agreeing not to harass the woman in the future. The facts clearly indicate that the plaintiff suffered personal injury (humiliation and damage to reputation), which can be traced to the challenged action. The federal court will likely hear the case because a genuine “controversy” is presented. Choice (C) is incorrect. A case is said to lack ripeness and, thus, not justiciable if the alleged injury is only speculative. Here, the woman has already suffered injury from her instructors because of their harassing remarks. Finally, choice (D) is incorrect. In orderfora federal court to review a case under Article Ill, there must be a “case or controversy.” That means that there must be some injury orthreat of an injury. Here, the woman is able to show that she has suffered injury from her instructors because of their harassing remarks.
27
Q
- Section 2022(a) of a state’s medical licensing code provides:“For the purposes of this statute, only those persons who have graduated from an optometry school located in the state and accredited by the board of optometrists shall be licensed to conduct the practice of optometry within the state.”A doctor graduated from a school of optometry located in a neighboring state. The following month, the doctor was granted a license to practice optometry in that neighboring state by its board of optometrists. For approximately one year, the doctor was engaged in the practice of optometry in the neighboring state. Subsequently, the doctor decided to relocate his practice to the state for which Section 2022(a) applies.The doctor re-domiciled in the state, where he opened a new office for the practice of optometry. When he initially opened his office, the doctor was unaware of the state’s licensing provision for optometrists. Since the doctor was a licensed optometrist for 10 years, he assumed that he could practice optometry in the state by reciprocity.The doctor received notification from the state’s board of optometrists that he was illegally practicing optometry and that he must immediately cease and desist from such practice.If the doctor challenges the constitutionality of Section 2022(a) of the medical licensing statute as violating the Commerce Clause, which of the following, if factually true, is the WEAKEST defense that the state could offer?(A) The statute will help to ensure that only the most qualified optometrists practice in the state.(B) The statute will help protect the optometrists in the state from competition outside the state.(C) The statute will help to ensure a continuously available number of practicing optometrists within the state.(D) The statute will help to ensure that the practicing optometrists in the state are subject to standards of a regulatory body.
A
- (B) The WEAKEST defense to the doctor’s constitutional challenge of the statute under the Commerce Clause is that the statute will help protect optometrists in the state from out-of-state competitors. State regulations, such as the medical licensing statute, that appear primarily to favor local economic interest as against outside competition are generally found to be unconstitutional as unreasonable burdens on interstate commerce. However, if the regulation is perceived as within a legitimate health and safety measure that cannot readily be achieved by less drastic means, a court will uphold even a regulation that is discriminatory. While a state may legislate to protect the health of its citizens, the statutory purpose regarding the licensing provision could be achieved by “reasonable nondiscriminatory alternatives, adequate to conserve the legitimate interests of the local optometrists.” Therefore, the statute could be challenged as a discriminatory burden on interstate commerce. Students should refer to Dean Milk Co. v. Madison, 340 U.S. 349 (1951), where the Supreme Court invalidated a Local health ordinance under the “unreasonable burdens” rule. Choice (A) is not the best answer. By favoring optometrists who are trained in the state, the state is in danger of presenting an undue burden on interstate commerce and, thus, violating the dormant commerce clause of Article I, Section 8. Under dormant commerce clause analysis, the only acceptable justification for such discrimination is safety or health. The state might be able to argue that its requirements, while discriminating against out-of-staters, are likely to produce optometrists who are better skilled and, thus, more likely to render service that will promote health and safety. Choice (C) is not the best choice. By favoring optometrists who are trained in the state, the state is in danger of presenting an undue burden on interstate commerce and, thus, violating the dormant commerce clause of Article I, Section 8. Under dormant commerce clause analysis, the only acceptable justification for such discrimination is safety or health. The state might be able to argue that its requirements, while discriminating against outof-staters, will produce a continuously available number of practicing optometrists within the state; hence, the state’s law will contribute to the visual health and perhaps even safety of the state’s residents. Choice (D) is not correct. By favoring optometrists who are trained in the state, the state is in danger of presenting an undue burden on interstate commerce and, thus, violating the dormant commerce clause of Article I, Section 8. Under dormant commerce clause analysis, the only acceptable justification for such discrimination is safety or health. The state might be able to argue that its requirements, while discriminating against out-of-staters, will produce a continuously available number of practicing optometrists within the state; hence, the state’s law will contribute to the visual health and perhaps even safety of the state’s residents.
28
Q
- A security guard relocated to a state which required all persons wishing to practice private security to have graduated from a security training program within the state. The security guard, having already graduated from a program in his former state, wishes to work again as a private security guard. He began to advertise his services but was informed by the state that he must cease and desist from working as a private security guard until he complied with state law and graduated from an in-state program.The security guard challenges the constitutionality of the licensing statute on grounds that it violates the due process clause of the Fourteenth Amendment. Which of the following statements is most accurate?(A) The security guard has the burden of persuasion to show that the state does not have a compelling state interest in enacting such legislation.(B) The security guard has the burden of persuasion to show that the denial of a license to work in private security violates his rights of due process.(C) The state has the burden of persuasion to show a compelling state interest in enacting such legislation.(D) The state has the burden of persuasion to show that the denial of a license to work in private security does not violate the petitioner’s rights of due process.
A
- (B) Since the security guard is challenging the constitutionality of the licensing statute, he has the burden of persuasion to show that the denial of the license violates his rights of due process. Upon such a showing, the burden then shifts to the state to prove that it has a compelling state interest in enacting such legislation. In In re Ruffalo, 390 U.S. 544 (1968), the Supreme Court held that if the government terminates an individual’s ability to engage in a profession, it must grant that individual a procedure to determine his fitness to be a member of the profession. Thus, an individual must be afforded a fair hearing before a government agency may revoke his license to practice a profession, such as, in this case, security. Choice (A) is incorrect. The security guard will first need to show that his due process rights have been violated when the state prohibited him from using his out of state private security license. If the security guard succeeds, the state will have the burden of showing that its licensing requirement furthers a compelling governmental interest. Choice (C) is incorrect. The security guard will first need to show that his due process rights were violated when the state prohibited him from using his out of state private security license. It is only after such a showing that the state bears the burden to prove that its denial furthered a compelling state interest. Choice (D) is incorrect. The security guard has the initial burden to show that the denial of his license violated his fundamental rights of due process. If the security guard is able to show this, the state then bears the burden of showing that its denial constitutes a compelling governmental interest and that the denial was the least restrictive means available to advance the compelling governmental interest.
29
Q
- A state recently enacted a statute requiring the following of all firms that do business with and for the state (defined generally as selling goods or providing services to the state, its agencies, or subdivisions):— Such businesses must purchase insurance only from insurance companies chartered in the state and thus be subject to regulation by the state insurance commissioner;— In hiring any unskilled laborers for employment in connection with state business, preference must be given to citizens of the state;— Nonresident aliens shall be prohibited from engaging in any state related business activities;— All buildings constructed for the state must have roofs composed of adobe, which is composed of yellow silt or clay deposits found only in the state.The proposed statute’s preamble recites that its provisions will ensure:— responsible insurance coverage for all those who do business with the state;— an increased standard of living for the citizens who comprise the state’s labor force;— the lowest possible expenditures by the state government; and— a beautiful, uniform aesthetic decor for all new state buildings.In evaluating the constitutionality of the state statute under the Supremacy Clause of Article VI, which of the following would be most relevant?(A) The number of aliens presently residing in the state.(B) The necessity for the enactment of this particular statute.(C) The treaties and immigration laws of the United States.(D) The overall unemployment rate in the United States.
A
- (C) Under the Supremacy Clause of Article VI, federal treaties are the supreme law of the land and supersede any state statutes which are in conflict therewith. According to the Supreme Court, Congress is vested under Article I, Section 8 with the primary authority to regulate immigration and aliens. Any state legislation that affects aliens is thus likely to be preempted (depending, however, in part on the activity regulated). Note that the Supreme Court has upheld a state law forbidding employment of illegal aliens under the state’s police power, since it did not interfere with the exclusive federal power over aliens. Choice (A) is incorrect. According to the Supreme Court, Congress is vested under Article I, Section 8 with the primary authority to regulate immigration and aliens. If Congress passes a law that is valid under this power to regulate immigration and aliens, the law, according to the Supremacy Clause, takes priority over any conflicting state laws. So, too, in some fields, such as immigration, Congress enjoys “field preemption” under the Supremacy Clause; under field preemption, any state statute that overlaps Congress’s field of regulation is also deemed unconstitutional. The number of aliens presently residing in the state is not relevant in determining whether the state statute conflicts with a congressional statute or whetherthere is field preemption. Choice (B) is incorrect. The necessity for the enactment of the state statute is mostly irrelevant in determining its constitutionality under the Supremacy Clause. Even if the state could show that its reasons for the statute were fabulous, the state could not save its statute from being preempted if the statute either conflicted with an otherwise constitutional federal statute or regulated in a field like immigration that was deemed by the Supreme Court to be an exclusively federal domain. Choice (D) is incorrect. The overall unemployment rate in the United States is mostly irrelevant in determining the constitutionality of the state statute under the Supremacy Clause. Much more important is to determine whether the state statute either violates an otherwise constitutional federal statute or whether the state statute regulates in a field like immigration that was deemed by the Supreme Court to be an exclusively federal domain.
30
Q
- A state recently enacted a statute requiring the following of all construction companies that do business with and for the state (defined generally as selling goods or providing services to the state, its agencies, or subdivisions):Such businesses must purchase insurance only from insurance companies chartered in the state and thus be subject to regulation by the state insurance commissioner;— In hiring any unskilled laborers for employment in connection with state business, preference must be given to citizens of the state;Nonresident aliens shall be prohibited from engaging in any state related business activities;— All buildings constructed for the state must have roofs composed of adobe, which is composed of yellow silt or clay deposits found only in the state.The proposed statute’s preamble recites that its provisions will ensure:— responsible insurance coverage for all those who do business with the state;an increased standard of living for the citizens who comprise the state’s labor force;— the lowest possible expenditures by the state government; and— a beautiful, uniform aesthetic decor for all new state buildings.If the state statute is attacked as violating the commerce clause, which of the following statements is most accurate?(A) The statute is a valid exercise of the state’s police power to legislate to protect the health, safety, morals, and welfare of its citizens.(B) The statute falls within the reserved powers under the Tenth Amendment.(C) The statute should be invalidated if there are nondiscriminatory reasonable alternatives available to serve legitimate local interests.(D) The statute is a valid exercise of state action only if the federal government has not previously “occupied the field” in this area.
A
- (C) If the state statute is challenged on commerce clause grounds, the court would apply the unreasonable burdens rule (also referred to as the least restrictive means test). State regulations, even in areas generally described as local, cannot be accepted if they impose an unreasonable burden on interstate commerce. In orderto determine the reasonableness of the burden, the court will balance the nature and extent of the burden that the state regulation would impose on interstate commerce against the purposes of the state regulation. Subsection (d) of the statute would place a discriminatory burden on producers of other building materials, both within and outside the state. Choice (A) is incorrect. Under commerce clause analysis, the court will determine whether a state’s benefits outweigh the burdens on interstate commerce. Here, Subsection (d) of the statute would place a discriminatory burden on producers of other building materials, both within and outside the state. Choice (B) is incorrect. The 10th Amendment contains the so-called police powers to regulate health, safety, welfare, and morals. The invocation of police powers, however, will be insufficient to help the state against the conclusion that it violates the dormant commerce clause. Choice (D) is incorrect. Actually, the state statute can still be held invalid for violating the commerce clause, irrespective of whether the state statute has occupied a field that is exclusive to the federal government. The Supremacy Clause of Article VI would invalidate local laws that conflict with the U.S. Constitution, including the commerce clauses therein.
31
Q
- A state has recently enacted a statute prohibiting the sale of beer in glass bottles. In accordance with the new law, all beer consumed within the state must be sold in aluminum cans that are recyclable. There is a provision of the statute that does permit breweries to distribute beer to bars, taverns, and restaurants in kegs for “on-site” consumption by patrons.Before the passage of the law, approximately 28% of all beer consumed in the state was packaged in glass bottles. Of that total, 75% of the beer was bottled outside the state, while 25% was bottled by companies in-state. The legislature passed the law at the strong urging of the aluminum can industry. It was estimated that one aluminum company alone spent over $5,000,000 in its lobbying campaign for the passage of the bill. Ironically, the new law even received strong support from environmentalists who believed that recyclable cans would help prevent littering and unsightly trash accumulation.The strongest federal constitutional argument against the validity of the law is that it violates the(A) equal protection clause of the Fourteenth Amendment by discriminating against beer bottlers.(B) privileges or immunities clause of the Fourteenth Amendment by preventing out-of-state beer bottlers from conducting their business in the state.(C) commerce clause by violating the negative implications on interstate commerce.(D) contracts clause by impairing the ability of beer bottlers to honor existing contracts for the sale of bottled beer in the state.
A
- (C) Although the federal commerce power is plenary, the states may also regulate commerce among themselves, subject to implications flowing from the Supremacy Clause. State regulation of interstate commerce must be reasonable and non-discriminatory, and the states may not interfere with the free flow of interstate commerce by favoring local interests or burdening out-of-state competition. The state statute, which prohibits the sale of beer in glass bottles, will effectively curtail outof-state production of 21% (75% of 28%) of all the beer consumed in the state. It is arguable that this large proportion is not only discriminatory against out-of-state bottlers, but unreasonable as well, since the urging of the aluminum can industry was the basis on which the state legislature passed the bill. Based on these facts, the strongest argument against the validity of the state statute would be that it vioLates the Commerce Clause and the negative implications flowing from it. Choice CD) is not the strongest answer because the Commerce Clause is a broader and stronger source of power than the contracts clause. Choice (A) is incorrect. The equal protection clause is not as strong an argument because even though the statute does arguably discriminate against out-of-state bottlers, the standard of review the court would use to determine its validity would be the rational-basis test, under which the state could prevail upon a showing of any rational basis for enacting the bill. Choice (B) is incorrect. Remember that the Privileges or Immunities Clause of 14th Amendment (as well as the Privileges and Immunities Clause of Article IV) only applies to U.S. citizens. A company is not a citizen.
32
Q
- A state has recently enacted a statute prohibiting the sale of computer printer ink cartridges in plastic bags. In accordance with the new law, all ink cartridges within the state must be sold in paper cartons that are recyclable.Before the passage of the law, approximately 28% of all ink cartridges used in the state was packaged in plastic bags. Of that total, 75% of the ink cartridges was packaged outside the state, while 25% was packaged by companies in-state. The legislature passed the bill at the strong urging of the paper industry. It was estimated that one paper company alone spent over $5,000,000 in its lobbying campaign for the passage of the bill. Ironically, the new law even received strong support from environmentalists who believed that recyclable paper would help prevent littering.Assume that the state Supreme Court adjudges the law to be unconstitutional on the grounds that it violates the contracts clauses of both the federal and the state Constitutions. The contracts clause of the state Constitution is similar to the one in the federal Constitution. The court so held because, in its judgment, the statute retroactively impairs the ability of plastic bag packagers to honor their existing contracts for the sale of ink cartridges packaged in plastic bags. The state attorney general now seeks review of this decision in the U.S. Supreme Court.How should the U.S. Supreme Court rule on this case?(A) Refuse to review this case on the merits because there is an adequate and independent state ground for the decision rendered below.(B) Reverse the decision on the merits with respect to the state constitutional issue because the federal constitutional holding rendered below makes such a state constitutional decision unnecessary.(C) Affirm the decision on the merits with respect to the federal constitutional issue and abstain from reviewing the state constitutional issue.(D) Affirm the decision on the merits with respect to both the federal and state constitutional issues because the state constitution must substantially conform with the federal constitution on this issue.
A
- (A) The Supreme Court may properly deny review of any matter that rests upon an adequate and independent state ground, since resolution of the “state” issue by the state court might preclude the need for federal review. As a general rule, the Supreme Court, upon reviewing a decision of a state court, reviews only the federal questions and not the state law questions [Nowak, Constitutional Law, p. 85]. Since the law was found by the state court to be unconstitutional as violative of the contracts clause of the state constitution, the Supreme Court would avoid review of the case on the merits because a separate and sufficient state law basis for unconstitutionality exists. Choice (A) is correct in stating this conclusion under the doctrine of adequate and independent state grounds. Choice (B) is incorrect as a misstatement of this doctrine, since the Supreme Court will decline to hear the case, not reverse the decision. Choices (C) and (D) reach the wrong conclusions because the Supreme Court will refuse review altogether, as long as any adequate state ground for state court review exists.
33
Q
- A state has recently released medical statistics showing that the number of new AIDS cases within the state has quadrupled from the preceding year. In 2008, the state reported that 2,250 people were diagnosed as being stricken with the AIDS virus. However, in 2009 the state confirmed that over 9,000 new persons contracted the deadly virus. In an effort to improve the health care of AIDS patients in the state, the Legislature has enacted a law providing public funds to assist privately owned hospitals.According to the law, every hospital in the state would receive $5,000 annually for each AIDS patient who was admitted to that hospital, and whose period of hospitalization exceeded one week. Although this bill was initially opposed by several churches and other organizations, the state’s legislature re-drafted the bill in a compromise effort to appease the opposition. In its final re-draft, the bill provided that the $5,000 annual subsidy “would not be paid to any hospital performing abortions.”Which of the following is the strongest argument against the constitutionality of the state’s statute?(A) The statute violates the establishment clause of the First Amendment, as incorporated into the Fourteenth Amendment, by adopting the controversial views of particular churches on abortion.(B) The statute violates the Fourteenth Amendment by conditioning the availability of public funds upon the recipient’s agreement to act in a way that makes more difficult the exercise by others of their fundamental constitutional rights.(C) The statute violates the equal protection clause of the Fourteenth Amendment by denying nonAIDS patients the same subsidy benefits as those received by AIDS patients.(D) The state will be unable to show it is advancing a legitimate governmental interest.
A
- (B) This fact situation is extremely representative of both the difficulty of Constitutional Law questions on the bar and of the closeness between long, and often similar, answer choices. The substantive guarantees of due process under the 14th Amendment require that legislation, to be constitutional, have a rational relationship to a legitimate end of government. In the area of fundamental rights, such as privacy (and including abortion), governmental power is limited to the extent that individuals may be afforded freedom of choice in matters relating to their personal life. The state statute limits the availability of funds for AIDS patients only to hospitals refusing to perform abortions. While the statute does not preclude individuals from having abortions performed, it does make more difficult the exercise of that right. Since legislation restricting fundamental rights is viewed under the strict scrutiny standard of review, the state then has a heavy burden to show that the measure is necessary to further a compelling interest. Choice (B) is the strongest argument presented and is, therefore, correct. Choice (A) is not the strongest answer. This is a potentially good choice, but it is uncertain whether the statute was motivated by religious views; the reference to churches in the facts are not conclusive. One could theoretically be opposed to abortion on strictly secular grounds. Choice (C) is incorrect. Under equal protection clause scrutiny, strict scrutiny will be applied to those laws that create suspect classes: race, ethnicity, nationality, and religion. Intermediate review will be applied to legal classifications based on gender and illegitimacy. All other legal classifications are subject to rational review, which is relatively easy for the state to pass. Here, non-AIDS patients will only be entitled to rational review under the equal protection clause. Finally, choice (D) is incorrect. The state statute will probably be subject to strict scrutiny because it is infringing a woman’s fundamental right to an abortion. Hence, the state will have to show that it is advancing something greater than a legitimate governmental interest; the state will have to show that it is advancing a compelling governmental interest.
34
Q
- In an effort to improve the health care of cancer patients, a state’s legislature has enacted a law providing public funds to assist privately owned hospitals. According to the law, every hospital in the state would receive $5,000 annually for each cancer patient who was admitted to that hospital, and whose period of hospitalization exceeded one week. Although this bill was initially opposed by several churches and other organizations, the state’s legislature re-drafted the bill in a compromise effort to appease the opposition. In its final re-draft, the bill provided that the $5,000 annual subsidy “would not be paid to any hospital performing abortions.”Which of the following is the strongest argument in support of the constitutionality of the state statute?(A) The Tenth Amendment reserves to the states plenary power over the allocation of their public funds.(B) Public subsidies in hospitals are privileges rather than rights and, therefore, are not entitlements protected by the due process clause of the Fourteenth Amendment.(C) The funding limitation in this statute does not directly prohibit or penalize the exercise of a fundamental right and is rationally related to the achievement of a legitimate state interest.(D) The statute promotes a compelling state interest in advancing the health, safety, and welfare of its citizenry.
A
- (C) Choice (A) is incorrect since the 10th Amendment’s reserved powers do not authorize plenary power to the states regarding allocation of public funds. Choice (B) is incorrect since the court would generally examine the purpose and circumstances underlying the authorization of public subsidies before mechanically applying the rights-privilege rationale. Choices (C) and (D) are both very persuasive. To determine which one is stronger, consider that the primary purpose of the statute itself is to provide public funding to hospitals for cancer patients, not to directly restrict abortion. This purpose promotes a legitimate interestwhich, under equal protection analysis, would be reviewed using the rational-basis standard, as stated in choice (C). Since choice (C) presents a federal constitutional basis of analysis, whereas choice (D) addresses merely a source of state power (i.e., police powers), choice (C) is preferred and, therefore, the correct answer.
35
Q
- A company operates passenger buses to all the major cities on the east coast. This service is authorized under a certificate of convenience and necessity issued by the Interstate Commerce Commission, pursuant to federal statute. The certificate does not, however, specify particular highways, streets, or locations for the bus service. The company’s advertising stresses that it picks up and delivers passengers at the center of each city that it serves. The company’s management regards this as a particularly effective advertising point in competition with the airlines and the railroads, because short-haul traffic supplies a major part of the bus company’s revenues.One of the major cities on the east coast, acting to relieve traffic congestion and air pollution, has recently enacted an ordinance that prohibits (a) the operation of all trucks and buses in a five- square- mile central business area, known as center city, between the hours of 10:00 A.M. and 4:00 P.M. on weekdays, and (b) all on-street parking of passenger automobiles in center city between the same weekday hours.The company’s bus station in this city is located in the heart of the center city area. According to its transportation schedules, more than 75 buses either enter or leave this station between the hours of 10:00 A.M. and 4:00 P.M. each weekday.If the company brings suit challenging the constitutionality of the city ordinance, the court will most likely declare the ordinance(A) constitutional, because it is within the city’s police power to regulate transportation services in the center city business district.(B) constitutional, because it is a valid exercise of municipal regulation in the area of intrastate commerce.(C) unconstitutional, because it violates the dormant commerce clause.(D) unconstitutional, because the ordinance is discriminatory per Se.
A
- (C) In all likelihood, the city ordinance prohibiting the operation of all trucks and buses within its center city business area between 10:00 A.M. and 4:00 P.M. would be violative of the dormant Commerce Clause, if less restrictive alternatives are available. Choice (A) is incorrect. It is true that the city council does have the police power under the 10th Amendment to regulate transportation within its jurisdiction because the state’s police powers of the 10th Amendment include the right to regulate matters concerning health, safety, morals, and welfare. However, the city council does not enjoy an unqualified right to exercise its police powers. Here, the city council appears to have violated the dormant Commerce Clause of Article I, Section 8 by presenting an undue burden on interstate commerce. Choice (B) is incorrect. The city council’s ordinance does regulate activities within its borders. But it also appears to present an undue burden on interstate commerce and, thus, will likely be deemed to violate the dormant Commerce Clause of Article I, Section 8. Choice (D) is incorrect. Actually, the law is not discriminatory, perse, since it affects intrastate commerce as well. However, the law does appear to present an undue burden on interstate commerce and will likely be deemed unconstitutional under the dormant Commerce Clause of Article I, Section 8.
36
Q
- A city ordinance provides that it shall be unlawful for any person to litter in a public park, and that a violation of this ordinance shall be punished by a fine of not more than $100 or imprisonment for not more than 30 days.A protestor organized a demonstration against federal governmental support to rebels in a foreign country. The rally took place in a city park, and attracted about 100 supporters. During the rally, the protestor delivered a speech to the attendees. At the conclusion of his speech, the protestor said, “I’m sick and tired of the garbage this administration is getting away with. Here’s what I think about its policy of rebel aid …“ At which point, the protestor walked over to a trash can and dumped its contents on the ground. As the crowd cheered wildly, the protestor shouted, “No more rebel aid … let’s stop this garbage now!” After littering the park, the protestor and his supporters left without picking up the trash.As a matter of constitutional law, may the protestor be prosecuted under the aforementioned city ordinance for littering the public park?(A) No, because littering the park in these circumstances could be construed as symbolic speech and, thus, it is protected from government regulation by the First and Fourteenth Amendments.(B) No, because the facts do not indicate that the protestor’s actions presented a clear and present danger that was likely to produce or incite imminent lawless action, thereby necessitating an abridgment of his freedom of speech.(C) Yes, because the city ordinance advances an important and legitimate public interest and is not directed at the suppression of communication.(D) Yes, because littering the park is conduct, not speech and, therefore, it may not be treated by the law as communication.
A
- (C) When the government regulates speech in a traditional public forum, it may only base its restriction on the content of the speech being regulated (1) if that content falls within a category of speech that the court has found unprotected by the 1st Amendment, or (2) if the government can demonstrate a compelling interest in suppressing the speech. However, the government may employ a time, place, or manner regulation to regulate speech in a traditional pubLic forum (streets, parks) so long as the regulation promotes an important interest unrelated to the suppression of a particular message and does not unnecessarily restrict the ability to communicate the message. Nowak, Constitutional Law pp. 975-76. In this question, the test maker is trying to trick students into thinking the Rosemont city ordinance regulates a protected 1st Amendment area. In fact, the littering ordinance regulates neither speech-related conduct nor speech-related content. The regulation promotes a legitimate objective by advancing a health and safety interest under the state’s police powers, and the ordinance passes 1st Amendment scrutiny since it is not directed at the suppression of communication. Choice (C) is correct. Choice (A) is incorrect. Even if the speech were considered as symbolic speech, it can still be regulated under the circumstances. The city has passed a proper time, place, and manner regulation that regulates the secondary effects of speech—not the speech itself—and provides reasonable alternatives for the protestor to say more or less the same thing without littering. Here, the city is advancing an important governmental interest in preventing littering (a secondary effect of the protestor’s speech), not deterring speech. Choice (B) is incorrect. In order to regulate the protestor’s littering, the city does not have to show that his speech constituted a clear and present danger. All that is required is that the city show it has passed a proper time, place, and manner regulation that regulates the secondary effects of speech—not the speech itself—and provides reasonable alternatives for the protestor to say more or less the same thing without littering. Here, the city is advancing an important governmental interest in preventing littering (a secondary effect of the protestor’s speech), not deterring speech. Choice (D) is incorrect. Littering might constitute symbolic speech since it is a combination of both conduct and speech. However, even if it were symbolic speech, the city is justified in regulating it because the city has used a proper time, place, and manner regulation that regulates the secondary effects of speech—not the speech itself—and provides reasonable alternatives for the protestor to say more or less the same thing without littering. Here, the city is advancing an important governmental interest in preventing littering (a secondary effect of protestor’s speech), not deterring speech.
37
Q
- A march was held to call attention to the fact that minorities are still disenfranchised from the electoral process. A congressional report revealed that in the certain parts of the country, only 42% of minorities eligible to vote were, in fact, registered. The report also indicated that certain states had residency laws restricting a person’s right to vote. As a consequence, Congress enacted a statute that makes any law denying minorities the right to vote shall be deemed unconstitutional.This federal statute will most likely be upheld under which of the following constitutional provisions?(A) Thirteenth Amendment.(B) Fourteenth Amendment.(C) Fifteenth Amendment.(D) Twentieth Amendment.
A
- (C) The 15th Amendment is a limitation prohibiting the states and the federal government from denying any citizen the right to vote on accountof race orcolor. Note, too, that the 15th Amendment has an “enabling clause” that allows Congress to enact legislation protecting against discrimination affecting the right to vote. Choice (A) is incorrect because the 13th Amendment provides that slavery shall not exist in the United States. Choice (B) is wrong because the 14th Amendment prohibits states from depriving any person of life, liberty, or property without due process and equal protection of the laws. Finally, choice (0) is incorrect because the 20th Amendment deals with presidential and congressional terms, not voting rights.
38
Q
- An illegal alien and her three children live in a U.S. city. The city council has enacted an ordinance requiring illegal aliens to pay a $100 “school fee” for each child enrolled in a city public school. Citizens and legal aliens are not required to pay the school fee.The city council has enacted this law to raise funds to hire additional teachers who are bilingual. The city determined that over 15% of children attending public schools in the city were illegal aliens. Furthermore, the city conducted a study and found that the overwhelming majority of illegal aliens residing in the city did not pay any local property taxes. As a result, since the city provided educational benefits to the children of illegal aliens, the school fee furthered a significant governmental interest.The illegal alien, who is indigent, is unable to pay the city school fee. The city will not allow her children to attend school unless the fee is paid. If she seeks your legal advice regarding the constitutionality of the fee, you should advise her that the ordinance is(A) valid, because the city can demonstrate that the school fee is necessary to further a compelling governmental interest.(B) valid, because the imposition of the school fee is substantially related to a legitimate governmental interest.(C) invalid, because denying educational services to children of undocumented aliens is not substantially related to an important governmental interest.(D) invalid, because the cost of educating the children imposed an undue burden on the public school system by requiring the school board to hire additional bilingual teachers.
A
- (C) In Plylerv. Doe, 457 U.S. 202 (1982), the Supreme Court held that illegal alien children are entitled to a free public education. In Plyler, the Court struck down a Texasstatute that (1) denied local school districts funds for education of illegal alienchildren, and (2) allowed school districts to deny a free public education to thesechildren. The majority of justices determined that the Equal Protection Clause ofthe 14th Amendment was intended to cover any person physically within a state’sborders regardless of the legality of his presence. Also, the Court rejected the notion that illegal aliens be treated as a “suspect” class. Instead, the Courtapplied “intermediate-level” scrutiny based on the following two factors: (1) the importance of public education, and (2) the powerless nature of the group. Thus, choice (C) is correct. Choice (A) is incorrect. The city probably will be unable to show that its school fee is necessary—the least restrictive means—to further a compelling governmental interest. In this instance, the city could have raised taxes for everyone, for example, to cover the cost for bilingual teachers. Choice (B) is incorrect. The problem with this answer is that the city’s school fee will be subject to strict scrutiny. The facts here are similar to those in Plyler v. Doe, 457 U.S. 202 (1982), where the Supreme Court rejected as unconstitutional, under the 14th Amendment’s Equal Protection Clause, a Texas statute that denied public education to children who were undocumented aliens. Plyler subjected Texas’s statute to strict scrutiny. The language of “substantially related” and “legitimate means” suggests a level of scrutiny below strict scrutiny. Choice CD) is incorrect. The language of “undue burden” generally comes from dormant commerce clause analysis. But, as phrased, the reference to undue burden in this answer does not involve any dormant commerce clause issues.
39
Q
- A state highway runs through the center of a city’s business district. As the highway passes through this business district, there is a stretch where the highway is too narrow to satisfS’ the safety standards for state roadways. Along this narrow stretch and on opposite sides of the street are located two businesses. Situated on the east side of the street is a bar and grill; directly across the street on the west side is a lounge.In order to comply with state highway regulations, the state highway department, acting pursuant to state statute, ordered the city to prohibit parking on one side of the highway within the business district. The city council convened a special meeting to determine whether parking should be prohibited on the east or west side of the street. During the meeting, the council was unable to resolve the issue. Finally, the city council president decided the only fair way to resolve the matter was to flip a coin. After the coin flip, the city prohibited parking at all times on the west side of the highway. On account of this decision, parking in front of the lounge was permanently prohibited. This caused the lounge to suffer a substantial decline in business because of the unavailability of on-street parking.As a consequence, the owner of the lounge brought suit for declarative relief against the city alleging that the parking ban on his side of the street was unconstitutional. In all likelihood, will the plaintiff be successful in this cause of action?(A) No, because as a governmental entity, the city enjoys immunity from such suits under the Eleventh Amendment.(B) No, because the city’s action was reasonable.(C) Yes, because the action of the city violated the equal protection clause of the Fourteenth Amendment.(D) Yes, because responsible government officials cannot conduct or formulate their decisionmaking processes by means of coin flips.
A
- (B) In dealing with the area of state economic regulation enacted so as to affect the health or safety of the citizenry, the Court will not strike down the state legislation if the benefit to health and safety outweighs the burden imposed on interstate commerce. The challenged legislation must pass a commerce clause test of “reasonableness” that is stricter than that used for due process and equal protection cases. Nowak, p. 271. Following the holding in South Carolina v. Barnwell, 303 U.S.177 (1938), where the Court upheld a width restriction on trucks using South Carolina state highways due to the safety concern arising from the state’s narrow roads, the city’s parking ban on one side of the highway due to the narrowness of the highway as it passes through the business district promotes a safety interest that is reasonable, and it will be upheld. Choice (A) is incorrect because the 11th Amendment generally prohibits people from suing the state for monetary damages. Here, the lounge is not suing the city for monetary damages. Choice (C) is incorrect. Under equal protection clause scrutiny, strict scrutiny will be applied to those laws that create suspect classes: race, ethnicity, nationality, and religion. Intermediate review will be applied to legal classifications based on gender and illegitimacy. All other legal classifications are subject to rational review, which is relatively easy for the state to pass. Here, the lounge does not constitute either a suspect class or quasi-suspect class. Thus, the city’s discrimination against the lounge, even if true, would only merit rational review under equal protection clause analysis; the city will easily pass such rational review. Choice (D) is incorrect. Under rational basis review, even something as silly as a coin flip may suffice under some circumstances where everything else is pretty much equal.
40
Q
- An industrial town with a population of 100,000, is located on the north side of a river. On the south side of the river is situated a rural community with a population of 40,000. For many years, various civic groups have urged that both communities merge into one township with a single governmental body. Independent studies have indicated that such a merger would result in an enormous tax savings to the residents of both municipalities by eliminating the duplication of services. On one previous occasion, proponents of the merger pian succeeded in having the proposal appear as an election referendum in each community. Although the merger referendum passed in the industrial town by a sizable margin, the voters of the rural community rejected the measure, fearing the combined government would be dominated and controlled by its neighbor’s larger representation.In order to alleviate the concern of the rural community’s voters regarding under representation in a merged governmental system, the respective city councils of both municipalities appointed a steering committee to formulate a new proposal. Accordingly, the steering committee devised a merger scheme wherein the city council of the united city, would consist of eight members. Within this proposed new system of government, each former municipality would be divided into four districts. With respect to the industrial town, each district would consist of 25,000 persons and each would have one elected city council member. By the same token, the rural community would be divided into four elective districts, each containing 10,000 residents. One city council member would be elected from each of these districts as well. The mayor would be elected at large by a popular vote of all residents in the newly created eight districts.Before this merger proposal was placed on the ballot, the state attorney general issued an advisory opinion stating that the measure did not, in her opinion, violate any statutory or constitutional provisions. Thereafter, the proposal was placed on the ballot and was overwhelmingly passed by the voters in both municipalities. After the election, but before the merger had officially been carried out, two taxpayers from the industrial town initiated suit to enjoin the unification, attacking the constitutionality of the disproportionate representative districts.Which of the following represents the plaintiffs’ strongest constitutional argument in support of their action?(A) The plaintiffs and other residents of the industrial town have been denied the equal protection of the law.(B) The plaintiffs and other residents of the industrial town have been denied the due process of the law.(C) The plaintiffs and other residents of the industrial town have been denied the privileges and immunities of citizenship as guaranteed by Article IV, Section 2.(D) The merged city would not constitute a republican form of government.
A
- (A) State control over the right to vote is not expressly limited by the federal Constitution. However, any inequality in allocating the right to vote based on using electoral districts established on criteria other than street population dilutes the “one man, one vote” principle and will be subject to strict scrutiny review by the Court. In Reynolds v. Sims, 377 U.S. 533 (1964), justice Warren formulated the one person, one vote rule: “If a State should provide that the votes of citizens in one part of the State should be given two times, or five times, or 10 times the weight of votes of citizens in another part of the State, it could hardly be contended that the right to vote of those residing in the disfavored areas had not been effectively diluted. The Equal Protection Clause requires that the seats in both houses of a bicameral state legislature must be apportioned on a population basis.” Nowak, Constitutional Law, p. 754. Choice (B) is incorrect. It seems unlikely that due process was denied to the plaintiffs given that the members of their city council voiced their concerns with the members of the other city council and a steering committee was formulated to represent the collective interests of both cities. Choice (C) is incorrect. The privileges and immunities clause of Article IV has been interpreted by the Supreme Court to prohibit discrimination by a city, even if the city’s policies discriminate against other in-staters, if the effect of the policy is to discriminate against out-ofstaters [United Building & Construction Trades Council v. Mayor and Council of Camden, 465 U.S. 208 (1984)]. However, in this instance, the aggrieved party consists entirely of in-staters. Choice (D) is incorrect. The Supreme Court faced a challenge to the use of statewide initiatives in Pacific States Telephone and Telegraph Company v. Oregon, 223 U.S. 118 (1912). In that case, the Court held that challenges to a state’s republican character are nonjusticiable political questions, and that the decision of whether a state is “republican” in conformance with the guarantee clause may be decided only by Congress, and the Court would not get involved.
41
Q
- Before a proposal to merge two towns was placed on a ballot, the state attorney general issued an advisory opinion stating that the measure did not, in her opinion, violate any statutory or constitutional provisions. Thereafter, the proposal was placed on the ballot and was overwhelmingly passed by the voters in both towns. After the election, but before the merger had officially been carried out, two taxpayers from one of the towns initiated suit to enjoin the unification, attacking the constitutionality of the proposal. The suit reached the state supreme court and was found to be constitutional under both state and federal law.The plaintiffs now file a motion seeking to have thiscase reviewed by the U.S. Supreme Court. The Court may(A) not hear the case, because it was decided on independent state grounds.(B) not hear the case, but may have it remanded to federal district court.(C) hear the federal issues involved, but decline to rule on the state issue.(D) rely on the advisory opinion rendered by the state attorney general and not hear the case on its merits.
A
- (A) Where a state court clearly holds that a state law violates (or does not violate) a state law or provision of the state constitution, that decision will be an adequate and independent state ground. This is true regardless of whether or not the opinion also decides that the state law violates a federal law as well. Indeed, where a state court holds that a state law violates both the state and federal constitutions, the doctrine of adequate state grounds will apply. This is precisely the case addressed by the question, because it is evident that the state court found the proposal constitutional under both state and federal law. Without more information (such as, for example, evidence that the opinion is based upon federal interpretation of a similar federal law), there is no reason to assume anything beyond the fact that the Supreme Court will decline the case, because the decision of the state court rests on an independent and adequate state ground. Choice (A) is thus correct. Choice (B) is incorrect, as it confuses the situation presented here with one where it is unclear whether the state made their decision based upon state or federal interpretations of statutes. In that situation, the Supreme Court may take the case, although it also has the power in that situation to dismiss or remand the case for clarification from the state court (not a federal district court). Choice (C) is incorrect because the Court will not hear the case exclusively on federal issues, because it will decline the case altogether, in light of the adequate state ground.
42
Q
- A federal grand jury was convened to investigate the alleged bribery by a foreign government of a special adviser to the President. The grand jury was probing the foreign government’s efforts to obtain delivery of transport planes, which were embargoed by the State Department. The Justice Department was trying to ascertain whether the foreign government had offered bribes to the special adviser and other members of the U.S. government in order to secure delivery of the transport planes.The special adviser testified before the grand jury that both he and the President had several conferences with the foreign government’s ambassador to the United States. He stated that during these meetings, they discussed problems in their region in general. He denied, however, any involvement in the foreign government’s efforts to secure delivery of the transport planes.Two weeks after the special adviser testified, the grand jury returned an indictment, charging him and two other members of the President’s staff with conspiracy to commit bribery and conspiracy to defraud the U.S. government. A special prosecutor was then appointed by the Justice Department to prepare the government’s case.Upon motion by the special prosecutor, a subpoena duces tecum was issued directing the President to produce the minutes of his meetings with the special adviser and ambassador of the foreign government. The special prosecutor was able to determine the exact dates of the meetings through appointment records that had been previously subpoenaed. The President released several edited transcripts of these conversations. On the same day, the President’s counsel filed a motion to quash the subpoena duces tecum, claiming an absolute executive privilege.Which of the following is the most accurate statement with regard to the President’s claim of executive privilege?(A) Under the separation of powers doctrine, the federal judiciary is without authority to review an assertion of executive privilege by the President.(B) The need for the confidentiality of high-level communications will sustain an absolute unqualified presidential privilege of immunity from judicial process on all occasions.(C) Article III does not vest the federal courts with power to resolve an intrabranch dispute.(D) Article II does not vest the President with an absolute, unqualified privilege to withhold evidence from a criminal prosecution.
A
- (D) Article II does not vest the President with an absolute, unqualified executive pri vilege as against a subpoena essential to enforcement of criminal statutes. To vest the President with such powers would upset the separation of powers doctrine and gravely impair the role of the courts under Article III. The Supreme Court, in United States v. Nixon, 418 U.S. 683 (1974), held that the courts have the power of judicial review and are the final arbiter ofa claim of executive privilege. Moreover, the Court concluded that the executive privilege may not be asserted as to subpoenaed materials sought for use in a criminal trial, since the Constitution requires the courts to adhere to the due process of law. Choice (A) is incorrect. In United States v. Nixon, 418 U.S. 683 (1974), the Supreme Court held that courts have ultimate authority for determining whether an invocation of executive privilege is justified. Otherwise, the executive branch would undermine the judiciary and thus violate the principle of the separation of powers. Choice (B) is incorrect. While confidentiality is obviously important for the executive branch, there is no unqualified immunity under the Constitution. In United States v. Nixon, 418 U.s. 683 (1974), the Supreme Court held that courts have ultimate authority for determining whether an invocation of executive privilege is justified. Otherwise, the executive branch would undermine the judiciary and thus violate the principle of the separation of powers. Finally, choice (C) is incorrect. Article Ill says: “The [federal] judicial Power shall extend to all Cases, in Law and Equity, arising under this Constitution, the Laws of the United States, and Treaties made, or which shall be made, under their Authority; to all Cases affecting Ambassadors, other public Ministers and Consuls.” Ambassadors and public ministers serve in the executive branch; the President himself is one such example. Thus, the federal courts may review the President.
43
Q
- Pursuant to a newly enacted statute, Congress created a nine-member agency empowered to promulgate rules governing air quality standards for the nation. In accordance with the statute, the President was authorized to appoint a majority of six members to the agency, and the other three positions were to be filled by the Senate.The nine members were duly appointed to the agency, and all appointees were subsequently approved in confirmation hearings. Thereafter, the agency issued the following regulations:(a) Requiring each motor vehicle operating in the United States to be equipped with a specified air/fuel control device;(b) Requiring each gas or oil furnace located in the United States to be fitted with a specified device to reduce emissions;(c) Requiring each State to establish and maintain a program under which each vehicle and each furnace shall be tested annually for compliance with federal emissions standards.Which of the following arguments would provide the strongest constitutional grounds against the authority of the statute?(A) The President does not have the constitutional power to appoint a majority of members to an administrative agency.(B) Congress does not have the executive authority to appoint members to an administrative agency.(C) An administrative agency does not have the constitutional authority to promulgate regulations that unduly burden interstate commerce.(D) An administrative agency does not have the constitutional authority to require states to supervise federal regulatory guidelines such as those enumerated in Subsection (c).
A
- (B) The power of the President to appoint and remove officers of the United States stems in part from express provisions of the Constitution and in part from the implications of express grants of power. Article II, Section 2, Clause 2 establishes in the President the power to appoint officers of the United States; it also provides that Congress may vest the appointment of inferior officers in either the President alone, in the courts, or in the heads of departments. At no time, however, may the legislative branch exercise executive authority by retaining the power to appoint those who will execute its laws. Thus, in Buckley v. Valeo, 424 U.S. 1 (1976), the U.S. Supreme Court held that Congress had violated Article II in allowing the President pro tern of the Senate and the Speaker of the House to appoint a majority of the voting members of the Federal Election Commission. Choice (A) is incorrect. Article II, Section 2 says that the President “shall nominate, and by and with the Advice and Consent of the Senate, shall appoint Ambassadors, other public Ministers and Consuls, Judges of the supreme Court, and all other Officers of the United States, whose Appointments are not herein otherwise provided for, and which shall be established by Law: but the Congress may by Law vest the Appointment of such inferior Officers, as they think proper, in the President alone, in the Courts of Law, or in the Heads of Departments.” Choice (C) is incorrect. This language of “undue burden” in regard to the commerce clause applies only to states and other local governments when their actions are analyzed under the dormant commerce clause. An administrative agency, acting as the federal government, can regulate commerce under the commerce clause, as long as the agency meets the requirements under Article I, Section 8 of the commerce clause. Choice (D) is incorrect. An administrative agency does have constitutional authority to require states to supervise federal guidelines regarding commerce, as long as the federal agency does not violate the state’s 10th Amendment rights against the federal government conscripting the state’s governmental resources [New York v. United States, 505 U.S. 144 (1992)].
44
Q
- Voters in a city passed a referendum legalizing gambling in a certain section of the city. The law established a commission, consisting of five individuals, that was in charge of licensing and overseeing the activities of the casinos. The members of the commission were to be appointed by the mayor. Pursuant to his statutory power, the mayor appointed four private citizens and a clergy member to the Commission.There is a constitutional challenge to the appointment of the clergy member as violating the establishment clause of the First Amendment. The mayor’s action is(A) unconstitutional, because the appointment of a clergy member to the commission fosters excessive governmental entanglement with religion.(B) unconstitutional on its face, because members of the commission are vested with enforcement powers.(C) constitutional, because the primary effect of appointing only one religious member to the commission does not, per Se, advance or inhibit religion.(D) constitutional, because commission membership is an appointive privilege and not an elective right.
A
- (C) Another issue commonly tested on the MBE deals with whetherreligious members are precluded from holding government offices. In McDaniel v. Paty, 435 U.S. 618 (1978), the Supreme Court declared unconstitutional a state law that prevented “Minister(s) of the GospeL, or priest(s) of any denomination whatever from serving as delegates to the state constitutional convention.” Interestingly enough, the disqualification of clergy members from legislative office existed in England and was followed by seven of the original states. The Supreme Court, however, found that this history was not decisive and invalidated the state Law. The majority of justices ruled that the free exercise of religion allowed members of the clergy to hold government office. Based on the holding in McDaniel, the appointment of a member of the clergy to a governmental agency or commission would not, perse, be violative of the establishment clause. Choice (A) is incorrect. In Lemon v. Kurtzman, 403 U.S. 602 (1971), the Court found that there was an establishment of religion if there was “excessive entangLement” between religion and government. For example, there was excessive entanglement if the government had to supervise a religious school to ensure that government subsidies for the teachers were not being used to teach religious subjects. Here, the clergy member is being hired to perform an avowedly religious purpose in maintaining good morals. Thus, no such ongoing supervision is required by the government. Choice (B) is incorrect. Under the 10th Amendment, a state—including local city agencies—is vested with police powers to regulate (and enforce) laws regarding health, safety, welfare, and morals. Choice (D) is incorrect. The distinction between privilege and right would not protect the government from violations against the Establishment Clause. If, say, the government gave $1 million to a church to spread religion, the government aid would clearly violate the Establishment Clause, even though it is a privilege, not a right, for the church to receive the money.
45
Q
- A state has enacted an abortion statute in an attempt to reconcile the conflicting interests involved when a woman chooses to terminate a pregnancy by abortion. The state’s statute provided that during the first trimester of pregnancy, a woman’s right to choose to terminate the pregnancy was paramount and could not be restricted in any manner. After the first trimester, the right of a woman to obtain an abortion was limited to cases where it was demonstrated by a physician that an abortion was necessary to protect the life or health of the woman seeking the abortion.In all likelihood, this abortion statute is(A) constitutional, because the state’s statutestrikes a proper balance between the fundamental right of a woman to choose to terminatea pregnancy by abortion and the due processright to life of the unborn child.(B) constitutional, because the state’s statute issubstantially related to the important stateinterest in protecting the health and life of themother.(C) unconstitutional, because the state’s statuteimposes an undue burden on the right to obtainan abortion.(D) unconstitutional, because it isirrational to impose virtually norestrictions on the right to obtain anabortion in the first trimester whileimposing significant restrictionson the right to obtain an abortionthereafter.
A
- (C) With respect to abortions, the trimester test of Roe has been partially overruled by Planned Parenthood of Southeastern Pennsylvania v. Casey, 505 U.S. 833 (1992). As a result of Casey, the state may restrict abortions so long as they do not place “undue burdens” on the woman’s right to choose. Here, the state’s statute does place an “undue burden” on the right to abortion because after the first trimester, a woman can only have an abortion to protect her health and life. Choice (A) is not the best answer because it utilizes the “fundamental” right to privacy language articulated in Roe v. Wade. However, the Court in Casey appeared to reject the Roe view that abortion was a “fundamental” right and restrictions are no longer to be strictlyscrutinized.Choice (B) is wrong because the Court applies the undue burden test to abortions, not an intermediate scrutiny standard as applied in gender discrimination. Choice (D) is incorrect. This option captures the general meaning of Planned Parenthood of Southeastern Pennsylvania v. Casey, 505 U.S. 833 (1992). There, the Supreme Court explained that a state is entitled to place more burdens on a woman’s right to an abortion as she progresses in her pregnancy. The rationale was that after viability, the state may seek to prohibit abortion to save the life of the fetus.
46
Q
- Congress recently enacted a statute that prohibits racial discrimination in the sale, transfer, or rental of real estate, either privately or publicly.Which of the following constitutional provisions would provide the best rationale for the enactment of this federal statute?(A) Under Article I, Congress has the power to enact laws that are “necessary and proper” to the general welfare.(B) The enforcement provision of Section 2 of the Thirteenth Amendment.(C) The enforcement provision of Section 5 of the Fourteenth Amendment.(D) The due process clause of the Fifth Amendment.
A
- (B) The 13th Amendment is unique in two respects. First, it contains an absolute bar to the existence of slavery or involuntary servitude; there is no requirement of “state action.” Thus, it is applicable to individuals as well as states. Second, like the 14th and 15th Amendments, it contains an enforcement clause enabling Congress to pass all necessary legislation. In this regard, the Court has held that the enforcement provision of the 13th Amendment has extended Section 1982 of the 1866 Civil Rights Act “to insure minorities the freedom to inherit, purchase, lease, sell, hold and convey real and personal property.” Most important, the 13th Amendment has been construed to prohibit both public and private racial discrimination in housing. Choice (A) is a potentially good option insofar as the necessary and proper clause can be invoked to further Congress’s powers under the commerce clause. But choice (B) is a better option. That is because the 13th Amendment provides an absolute bar to the existence of slavery or involuntary servitude, whether committed by the government or private individuals, and there is a clause in the 13th Amendment that permits Congress to pass all appropriate legislation to enforce the 13th Amendment. Choice (C) is incorrect. Remember that the 14th Amendment only protects a person from the actions of local governments, not private actors. Thus, the 14th Amendment will not help Congress to prevent racial discrimination by private individuals. Finally, choice (D) is incorrect. Like the 14th Amendment, the 5th Amendment only protects a person from discrimination by the federal government. Thus, the 5th Amendment will not help Congress to prevent racial discrimination by private individuals.
47
Q
- A staff assistant for a state agency was convicted in federal court of taking bribes from a foreign government for the purpose of influencing an upcoming vote on a waterworks bill. He was sentenced to probation. The staff assistant had served in the agency long enough to become fully qualified for his pension upon retirement under the terms of an agreement between the pensions board and the union. The staff assistant retired and immediately started receiving monthly pension checks.Subsequently, the governor signed into law an act, which provided in part:“Section 8. Any member of a state agency’s staff who is convicted of… bribery … shall not beentitled to receive any retirement or other benefit or payment of any kind from the state … Such conviction shall be considered a breach of the staff member’s employment contract.”The staff assistant received a letter from the state which stated that pursuant to this new act the state is immediately discontinuing pension benefit payments to you on account of your bribery conviction.The staff assistant contacted an attorney, who challenged the discontinuance of benefits on the grounds that the new law was unconstitutional.To counter one of the attorney’s possible arguments regarding the unconstitutionality of Section 8 of the act, the state’s best rebuttal would be that(A) the staff assistant was afforded an opportunity to express his views about the new legislation at public hearings, prior to the enactment of the statute.(B) deprivation of pension benefits is not cruel and unusual punishment.(C) a letter sent through ordinary mail is sufficient notice to satisfj due process for discontinuation of pension benefits.(D) it is implicit that one of the conditions of the state’s contract of employment with a state agency staff member is that he shall not engage in bribery.
A
- (D) This question requires a two-step analysis to select the best alternative. First, you must determine the constitutional issue involved and then, second, apply the appropriate constitutional principle to the question asked, i.e., the state’s best rebuttal to a constitutional challenge to the pension forfeiture statute. Choice (D) is correct since the argument concerning a condition of employment contract with a agency staff member affects the Contract Clause of the Constitution. The staff member’s attorney, could validly challenge the constitutionality of the statute, alleging unconstitutional impairment of the obligation to contract. The pension forfeiture statute would be violative of the Contract Clause since under the circumstances, the staff member has satisfied the conditions of retirement eligibility (he became fully qualified for his pension on retirement, as stated in the facts). His retirement pay has ripened into a full contractual obligation and has become a vested right. Therefore, the pension forfeiture statute would be an unconstitutional impairment of his vested right to receive retirement benefits (his pension). Choices (A) and (B) are irrelevant to the constitutional issues involved. Answer (C) is inapplicable, since notice is not an issue here.
48
Q
- A state’s attorney was convicted in federal court of taking bribes to drop charges against an oil company. He was sentenced to probation. The state’s attorney had served long enough to become fully qualified for his pension upon retirement. The state’s attorney retired and immediately started receiving monthly pension checks.Subsequently, the governor signed into law a statute which provided in part:“Section 8. Any member of the state attorneys staff who is convicted of… bribery … shall not beentitled to receive any retirement or other benefit or payment of any kind from the state . Such conviction shall be considered a breach of the staff member’s employment contract.”Following the enactment of the statute, the state’s attorney received a letter from the state which stated they are immediately discontinuing pension benefit payments to him on account of his bribery conviction.The state’s attorney contacted an attorney, who challenged the discontinuance of benefits on the grounds that the new law was unconstitutional.In order to reinstate the state attorney’s pension on the grounds that the statute is unconstitutional, the attorney’s strongest argument would be that(A) the state attorney was retroactively punished.(B) the statute is an ex post facto law(C) the supremacy clause invalidates the state law, because there is federal legislation regulating pension and profit sharing plans.(D) the statute has a chilling effecton legislators’ rights to freely discuss pending bills with members of their staff.
A
- (B) The two ex post facto clauses in the U.S. Constitution prohibit Congress and the state legislatures from enacting laws that have a retroactive effect. The statute is an example of an expost facto law that renders an act punishable in a manner in which it was not punishable when committed under the facts presented (since the state’s attorney qualified for pension benefits before the pension forfeiture statute was enacted). Thus, the statute, which denied the state’s attorney a pension because of the bribery conviction during his employment applied retroactively to the state’s attorney. Hence, the pension forfeiture statute would be held violative of the expost facto clauses of the U.S. Constitution. Be aware that although choice (A) is a correct statement of fact, choice (B) is preferred, because it is the correct statement of law. When you are confronted with correct statements of fact and law, the latter is the preferred alternative. Choice (C) is incorrect because even if this were factually true, there would not necessarily be any violation of the Supremacy Clause unless the federal and state laws conflicted with each other or the state legislation regulated in a field that was reserved exclusively for the federal government. Neither appears to be the case. Choice (D) is incorrect. Usually, statutes are deemed to violate the 1st Amendment if they are so ambiguous as to scare people—”chill them”—into forgoing their speech. That does not appear to be the situation in our example.
49
Q
- A state trooper was convicted in federal court in 2008 of taking bribes. He was sentenced to probation. He had served as a state trooper since 1978, and in 1998, he became fully qualified for his pension upon retirement. In 2006, the trooper retired and immediately started receiving monthly state pension checks.In 2009, the governor of the state signed into law a statute, which provided in part:“Section 8. Any member of the state law enforcement staff… who is convicted of… briberyshall not be entitled to receive any retirement or other benefit or payment of any kind from the stateSuch conviction shall be considered a breach of the staff member’s employment contractSection 12. The provisions of this Act shall be retroactive to December 31, 1975.Section 14. This Act shall take effect immediately.”Following the enactment of the statute, the state trooper received a letter from the state which stated that pursuant to the statute, the state is immediately discontinuing pension benefit payments to you on account of your 2008 bribery conviction.”The trooper contacted an attorney, who challenged the discontinuance of benefits on the grounds that the new law was unconstitutional.Assume that the state’s highest court holds the statute constitutional. The trooper might still be able to eventually have his pension reinstated if(A) he exercises his constitutional right to discretionary review in the U.S. Supreme Court.(B) he receives a presidential pardon for his bribery offense.(C) he can show that he was convicted before the effective date of thestatute.(D) he can show that the statute violates the dormant commerce clause.
A
- (B) Of the four choices listed, only (B) provides a correct statement regarding the possibility of the reinstatement of the trooper’s pension. Article II empowers the President to grant reprieves and pardons for offenses against the United States. The President may pardon absolutely or conditionally commute sentences, and remit fines, penalties, and forfeitures. The facts state that the trooper was convicted of bribery in federal court. In all likelihood, therefore, he was being prosecuted for a federal criminal offense. Choice (A) is incorrect since he does not have a constitutional right to discretionary review in the U.S. Supreme Court. Such right of appeal to the U.S. Supreme Court is not a constitutional right, but a statutory right; Congress has enacted legislation regarding the appellate review of the Supreme Court. Choice (C) is incorrect because even if this were factually true, it would not make any legal difference because the State is not punishing him and, thus, the State is not violating the prohibition against ex post facto laws in Article I, Section 9. Choice (D) is wrong because there is nothing on these facts to suggest that the statute places a burden on interstate commerce.
50
Q
- Within the last two years, the number of cases coming before the U.S. Supreme Court has quadrupled. Because of this increased work load, the Court has complained that it is unable to properly review all of its cases. As a consequence, Congress formed a committee to conduct a study on improving the functioning and operation of the Court. Based on the committee’s recommendations, Congress enacted a statute dividing the Court into two panels. One panel would be assigned to handle criminal cases exclusively, while the other panel would handle all non-criminal matters. Each panel would be composed of four associate justices and a chief justice. According to the new law, the decisions of each panel would be final and not reviewable by any other court or judiciary.Which of the following is the strongest argument against the constitutionality of this federal statute?(A) The statute contravenes the requirement in the Constitution that there be one Supreme Court.(B) The statute does not fall within the enumerated powers of Congress and is not necessary and proper for the effectuation of those powers.(C) Based on the doctrine ofjudicial supremacy, Congress does not have authority to legislate with respect to the jurisdiction of the Supreme Court.(D) Based on the separate sovereignty doctrine, Congress does not have authority to interfere with the procedural machinery of the Supreme Court.
A
- (A) The Supreme Court is the only federal court created directly by the Constitution. Article III, Section 1 mandates that judicial power be vested in “one Supreme Court.” See Nowak, Constitutional Law, p. 24. Therefore, the Congressional statute to divide the Supreme Court into two panels is unconstitutional since it contravenesArticle III of the Constitution. Choice (B) is factually true, but its rationale is not as precise as choice (A). Students should be aware that Article III, Section 1 vests judicial power as to the inferior courts—including federal district courts and courts of appeal only “as the Congress may from time to time ordain and establish.” This plenary power of Congress includes not only the establishment of such courts, but also the authorization of their jurisdiction, the power to remove jurisdiction of certain classes of cases, and the power to terminate the courts’ existence. Article Ill judges are appointed for life, and their compensation may not be diminished during their term in office. Choice (C) is incorrect. The concept of “judicial supremacy” finds no explicit mention in the Constitution. In Marbury v. Madison, 5 U.S. (Cranch 1) 137 (1803), the Court stated that it is emphatically the province of the judicial branch to say what the law means. Yet this, in itself, does not explain why the congressional statute in the example is unconstitutional. A better explanation is that Article Ill, Section 1 mandates that judicial power be vested in “one Supreme Court.” Therefore, the Congressional statute to divide the Supreme Court into two panels is unconstitutional since it contravenes Article III of the Constitution. Choice (D) is incorrect. The separate sovereignty doctrine means that different powers of jurisdiction belong to different states and that the federal government’s sovereignty differs from that of the state governments. For example, a person could be prosecuted forthe same crime under both federal and state laws without violating the constitutional prohibition against double jeopardy. Congress and the Supreme Court are governed less by “separate sovereignty doctrine” than by the principle of the separation of powers.
51
Q
- The President appointed a delegation to enter into negotiations with representatives of a foreign government to study the problem of preventing the extinction of certain species of seals. The delegation’s goal was twofold: to study the problem and to formulate regulations in a bilateral agreement that would protect the endangered species and provide for a permanent commission that would continually monitor enforcement of the proposed regulations. After compiling their findings and drafting the necessary regulations, the President and the leader of the foreign government entered into a treaty to form a permanent commission to oversee the problem and to grant it the necessary enforcement powers.The validity of this treaty would most likely be upheld under which of the following principles?(A) The presidential power to conduct foreign affairs.(B) An ancillary power of the President under his treaty-making power.(C) The treaty-making power, but only if the treaty is ratified by two-thirds of the Senate.(D) The treaty-making power, but only if the treaty is ratified by a majority in Congress.
A
- (C) Under Article II, Section 2, the only constitutional limitation upon the President’s power “to conduct foreign affairs” is with regard to treaties, which become valid only when ratified by two-thirds of the Senate. Choice (A) is not a bad answer, but a better option is available in Choice (C), given that there is explicit constitutional support in Article II, Section 2 for the President’s power to make treaties, whereas there is no explicit mention of the President’s power to conduct “foreign affairs”. Besides, the President must, under Article II, Section 2, receive ratification by two- thirds of the Senate to complete a treaty; the president cannot rely on his “foreign affairs” powers to do that. Choice (B) is incorrect. The President cannot rely on his ancillary powers under his broader treaty-making power. He does, however, have to rely on the Senate, as Article II, Section 2 requires that he receive ratification by two-thirds of the Senate to complete a treaty. Finally, choice (D) is incorrect. Article II, Section 2 requires that the President receive ratification by two-thirds of the Senate to complete a treaty.
52
Q
- The President appointed a delegation to enter into negotiations with representatives of a foreign government to study the problem of preventing the extinction of certain species of rabbits. The delegation’s goal was twofold: to study the problem and to formulate regulations in a bilateral agreement that would protect the endangered species and provide for a permanent commission that would continually monitor enforcement of the proposed regulations. After compiling their findings and drafting the necessary regulations, the President and the leader of the foreign government entered into a treaty to form a permanent commission to oversee the problem and to grant it the necessary enforcement powers.Assume that after the treaty goes into effect, a state legislature enacts a statute that provides that “any licensed rabbit-hunter in the state and its surrounding environs may increase his monthly catch of rabbits from 10 to 15 in each of the specified months of the authorized rabbit-hunting season from the first day of October until the last day of February.” If challenged, the enactment of the aforementioned statute would most likely be declared(A) constitutional, because the regulation of hunting is within the area of state action.(B) constitutional, because the enactment falls within the Tenth Amendment’s reservedpowers.(C) unconstitutional, because it violates the commerce clause.(D) unconstitutional, because all treaties are the supreme law of the land.
A
- (D) UnderArticleVi, Paragraph 2, alL treaties “which shall be made underthe authorityof the U.S.” are the “Supreme law of the land” (along with the Constitution itselfand laws of the U.S. made in pursuance thereof). As a consequence, it is clear thatany state action in conflict with a treaty is invalid. Choice (A) is incorrect. Even ifthere is state action in this example (and there does appear to be), that alone wouldnot make the statute constitutional. In fact, state action simply means that there isgovernmental involvement. Choice (B) is incorrect. As a matter of law, this is truebecause of the state’s police powers under the 10th Amendment to regulate health,safety, welfare, and morals. However, in this instance, the state statute conflictswith the treaty. Under Article VI, Paragraph 2, aLl treaties “which shall be madeunder the authority of the U.S.” are the “Supreme law of the land” (along with theConstitution itself and laws of the U.S. made in pursuance thereof). As a consequence, it is clear that any state action in conflict with a treaty is invalid. Choice (C)is not a bad answer, since the state’s actions would seem to have some effect oncommerce between the state and the foreign government, and under Article I, Section 8, only Congress has the authority to regulate commerce with foreign nations.But a better answer is available in choice (D) because there is explicit and directsupport for it in the Constitution.
53
Q
- During a three-month period, a city was stunned by a series of mysterious deaths, which claimed the lives of 20 people. Although all the victims had apparently died from poisoning, the police were in a quandary as to who was responsible for the killings. There was finally a breakthrough in the investigation when a police toxicologist determined that all the victims had died after eating poisoned apples which had been purchased at a local market. The apples had all been contaminated with a pesticide. The police then received a letter from a person who claimed responsibility for the poisonings. The letter stated that the killings were in retaliation for the city’s new policy of prosecuting toxic polluters.Acting upon an anonymous tip, the police arrested the owner of a pest control company engaged in the manufacture of pesticides, and charged him with 20 counts of murder. Thereafter, the city’s largest newspaper ran a series of articles on the killings and referred to the owner as the pesticide poisoner. After the preliminary hearing, the state trial judge issued an exparte injunction against the newspaper prohibiting it from publishing any news during the trial that might be prejudicial to the owner. The newspaper appealed.In light of the U.S. Supreme Court cases to date, the state appellate court should(A) dissolve the injunction, because a news story about a matter of public interest is absolutely privileged.(B) dissolve the injunction, because it is an impermissible prior restraint on the freedom of the press.(C) uphold the injunction, because the inference of guilt in any prejudicial coverage would deny the defendant his constitutional right of a fair trial.(D) uphold the injunction, because reference to the defendant by the name pesticide poisoner would be inflammatory and prejudicial.
A
- (A) or (B) According to Justice Berger, “the compatibility of a commitment to an uninhibited, robust, and wide-open discussion of public issues in a free press with a commitment to a criminal process in which the conclusions to be reached in a case will be induced only by evidence and argument in open court has been the subject of long standing debate.” See Nebraska Press Association v. Stuart, 427 U.S. 539 (1976). In this area, the rights of the press often conflict with the rights of the accused. AccordingLy, the Supreme Court offered a qualified response to this question when it invalidated a Nebraska District Court “gag order” that prohibited the press from the publication of certain implicative evidence pertaining to a murder suspect until the jury selection process was completed. Based on the Nebraska decision, choices (A) and (B) would both be arguably correct. Choices (C) and CD) are incorrect because these would not necessarily prejudice the defendant’s right to a fair trial because it does not suggest a connection between the owner and the killings.
54
Q
- A defendant was charged with murder. The killing took place on February 1. At the time that the crime occurred, this jurisdiction required a unanimous verdict of all 12 jury members for a conviction on a capital offense. On November 1, the state legislature enacted a new law requiring a majority vote of seven of 12 jurors to convict on a capital offense. The defendant’s trial began on December 1. He was subsequently convicted of murder by an eight- to-four vote. Following the trial, the defendant’s attorney filed a motion to set aside the verdict.Which of the following would provide the strongest constitutional grounds to overturn the verdict?(A) The ex post facto clause.(B) The contracts clause.(C) The due process clause of the Fourteenth Amendment.(D) The Sixth Amendment right to a fair trial.
A
- (A) The ex post facto clause forbids both the states and the federal government from enacting retroactive criminal laws. The most common sort of ex post facto law is one that creates a new crime and applies it retroactively to conduct not criminal at the time it was committed. In addition, the ex post facto clause prohibits the retroactive application of an increase in the punishment for a crime that carried a lesser penalty when committed. Another aspect of the ex post facto prohibition is concern with retroactive changes in evidence and procedure that operate to the disadvantage of the criminal defendant by making conviction easier. Thus, a statute that changes the burden of proof on the prosecution from the usual rule of beyond a reasonable doubt to one of the preponderance of the evidence is ex post facto if retroactive. By analogy, in this hypo we have a situation where the legislature changed the unanimity jury verdict requirement for capital offenses afterthe defendant was arrested and charged with murder. As such, choice (A) is correct. Choice (B) is incorrect. Article I, Section 10, the so-called “Contracts Clause,” states that no state shall pass any “Law impairing the Obligation of Contracts.” There are no contracts at issue in this example. Choice (C) is incorrect because the defendant wilL receive due process of law insofar as he will be given a trial and afforded the protections thereof. Choice (D) is not a bad answer, but the defendant will receive a fair trial insofar as he has been guaranteed a jury as required by Article III, Section 2.
55
Q
- The Pentagon has recently released a civil defense plan in the event of nuclear war. According to the Pentagon’s study, certain essential citizens would be evacuated once it was determined that a nuclear war was imminent. Essential citizens would include scientists, carpenters, and the young. The study also recommended that certain non-essential citizens such as the elderly, the infirm, and persons in penal institutions not be evacuated since their future contributions would be less important in the rebuilding of the country following a nuclear war.An employee of the Pentagon, was instructed to conduct a public opinion survey regarding the controversial plan. Pentagon officials directed the employee to interview citizens in a door-to- door canvass to determine public opinion for the civil defense plan. After the employee conducted his door-to-door interview canvassing, he was prosecuted for not obtaining prior consent of the citizens he interviewed.The employee’s strongest argument is that the prosecution(A) violates his right to free speech.(B) violates the intergovernmental immunity of a federal employee.(C) deprives him of his employment interest without due process.(D) impairs the obligation of his employment contract.
A
- (B) Although a city ordinance may prohibit the business practice of soliciting magazinesubscriptions door-to-door without prior invitation of the homeowner, in Beard v.Alexandria, 341 U.S. 622 (1951), the Court specifically relied on the commercialnature of the transactions in question. On the other hand, in Martin v. Struthers,319 U.S. 141 (1943), the Court held an ordinance invalid that forbade any personto knock on doors, ring doorbells, or otherwise summon any residents to the dooras violative of the freedom of speech and press. In this regard, the substantive guarantee of due process requires that legislation have a rational relationship to theLegitimate ends of government. If a law does not have such a relationship, it wouldbe an unconstitutional deprivation of liberty as to those persons affected. Here,the employee’s strongest argument is that the prosecution violates the intergovernmental immunity of a federal employee. Note that the employee was performingessentiaLly a governmental, not a proprietary, function (in the door-to-door canvassing). Thus, choice (B) is the best answer. Choice (A) is incorrect. Although a cityordinance may prohibit the business practice of soliciting magazine subscriptionsdoor-to-door without prior invitation of the homeowner, in Beard v. Alexandria, 341U.S. 622 (1951), the Court specifically relied on the commercial nature of the transactions in question. On the other hand, in Martin v. Struthers, 319 U.S. 141 (1943),the Court held an ordinance invalid that forbade any person to knock on doors, ringdoorbells, or otherwise summon any residents to the door as violative of the freedom of speech and press. But choice (B) is the best answer because the employeewas not exercising his free speech rights as a private citizen but as an employee ofthe federal government. Choice (C) is incorrect, the employee was working for thefederal government and, thus, was performing a governmental, not a proprietary,function. Accordingly, the due process clause is not the best answer. FinalLy, choice(D) is incorrect. Article I, Section 10, the so-called “Contracts CLause,” states that no state shall pass any “Law impairing the Obligation of Contracts.” However, the prosecution does not impair the employee’s obligation because he could have conducted the survey without going door-to-door.
56
Q
- A state is concerned with the increase in teenage use of alcohol. In an effort to decrease exposure to alcohol, which poses harmful health risk, the state legislature has enacted a statute to restrict various methods of advertising by alcohol manufacturers. One of the provisions of the law states that advertising of alcohol prices is not permitted except by placement of a sticker on the bottle or container. An alcohol company who was a major distributor of alcohol in the state, claims the advertising restriction violates its constitutional rights protected by the First and Fourteenth Amendments.If the alcohol company files suit challenging the validity of the state statute, the court should rule the statute(A) constitutional, because the state law is rationally related to the health and safety of the state’s citizens.(B) constitutional, because the restriction on commercial speech directly advances a substantial government interest.(C) unconstitutional, because the regulation on commercial speech is not necessary to further an important government interest.(D) unconstitutional, because the state could achieve its objective by a less restrictive means.
A
- (D) Quite often, Multistate Constitutional Law questions are based upon case precedent. This question, for example, is based upon the ruling in 44 Liquormart v. Rhode Island, 517 U.S. 484 (1996), in which a Rhode Island statute prohibited all advertising of liquor prices, except for price tags displayed with the merchandise. The Supreme Court invalidated the law because regulations of commercial speech must be “narrowly tailored” and should be no more extensive than is necessary. In both 44 Liquormartand in this question, the state is attempting to prohibit the dissemination of truthful, nonmisleading advertising. Choice (D) is correct because a state will not be permitted to completely ban commercial advertising but must use a means narrowly tailored to achieve the desired objective. Choice (A) is incorrect. Commercial speech, which is what we have in our example, may not get as much protection as political speech, but it may be entitled to more than you might think. Under 44 Liquormart v. Rhode Island, 517 U.S. 484 (1996), the Court rejected a state statute that prohibited all advertising of liquor prices, except for price tags displayed with the merchandise. In doing so, the Court subjected the statute to strict scrutiny. The facts in our example look similar to 44 Liquormart insofar as our example also contains a statute that prohibits nearly all advertising of alcohol. But choice (A) suggests in its reference to “rationally related” that the Court will use rational review, not strict scrutiny. Choice (B) is incorrect. Under strict scrutiny, the court will ask whether the statute furthers a compelling government interest (although sometimes the Court will use the language of “substantial” government interest). The statute’s concern for health would certainly seem to be a compelling (or substantial) government interest, but the statute probably will be unable to satisfy the second part of strict scrutiny: are the means chosen “narrowly tailored”? In other words, are there less-restrictive alternatives? Choice (C) is incorrect. The statute’s concern for health would certainly seem to be a compelling (or substantial or important) government interest.
57
Q
- An American franchise operates as an importer and distributor of bicycles manufactured by a foreign country, and maintains several warehouses for the wholesale distribution of the bicycles. A warehouse located in a county handles the distribution of bicycles for several states. The bikes and bike tires are shipped separately to the county warehouse. The tires are mingled, stacked, and stored in the warehouse along with various other tires. The bicycles, on the other hand, arrive completely assembled in their shipping crates and remain on the loading docks. The county imposes an ad valorem property tax on the bikes and tires.The county’s ad valorem tax may properly be assessed against the(A) tires only.(B) bicycles only.(C) tires and bicycles.(D) neither the tires nor the bicycles.
A
- (A) Article I, Section 10, Clause 2, of the U.S. Constitution provides that “No State shallwithout the consent of Congress, lay any Imposts or Duties on Imports or Exports,except what may be absolutely necessary for executing its Inspection Laws.”In Michelin Tire Corp. v. W. L. Wages Tax Comm., 423 U.S. 276 (1976), the U.S.Supreme Court held that while tubes in their corrugated shipping cartons wereimmune from ad valorem taxation, the tires lost their status as imports and becamesubject to taxation because they had been mingled with other tires imported inbulk, sorted, and arranged for sale. Similarly, in this hypo, the bikes were immunefrom the tax, since they remained “imports” in transit, whereas the tires lost their“import status” once they became part of the tire inventory at the distribution warehouse. Therefore, choices (B), (C), and (D) are aLl incorrect under this analysis.
58
Q
- An American company operates as an importer and distributor of guitars manufactured by a foreign company, and maintains several warehouses throughout the U.S. for the wholesale distribution of the guitars. A warehouse located in a city handles the distribution of guitars for several state areas. The guitars and guitar strings are shipped separately to the city warehouse. The guitar strings are mingled, stacked, and stored in the warehouse along with various other guitar strings. The guitars, on the other hand, arrive completely assembled in their shipping crates and remain on the loading docks. The city imposes an ad valorem property tax on the guitars and strings.The power of the city to impose an ad valorem tax on the foreign company’s guitars and/or strings would most likely be upheld under(A) the commerce clause.(B) the reserved power of the states as granted under the Tenth Amendment.(C) the necessary and proper clause. (D) the import and export clause.
A
- (B) The state’s power to enact such property taxes is derived from the 10th Amendment’s reserved powers. All other choices are incorrect, since they reflect powers of the federal government. Choice (A) is incorrect. Under Article I, Section 8, only Congress has the authority to exercise commerce clause powers. Choice (C) is incorrect. Under Article I, Section 8, only Congress has the authority to exercise powers under the necessary and proper clause. Choice (D) is incorrect. Article I, Section 10 contains the import and export clause. This clause prohibits the states: “No State shall without the consent of Congress, lay any Imposts or Duties on Imports or Exports, except what may be absolutely necessary for executing its Inspection Laws.” The clause, therefore, does not bestow any powers upon the states.
59
Q
- A pharmaceutical company manufactured a new contraceptive sponge. The pharmaceutical company initially intended for the contraceptive to be available for purchase without a doctor’s prescription. However, a study by the Food and Drug Administration revealed that the contraceptive might prove harmful to some users with preexisting health problems. As a result, Congress enacted legislation prohibiting the shipment and sale of the contraceptive across state lines to those who do not have a doctor’s prescription.This law is probably(A) constitutional, because Congress has the power to provide for the general welfare.(B) constitutional, because Congresshas the power to regulate interstatecommerce.(C) unconstitutional, because it deprives the manufacturer a property right without just compensation.(D) unconstitutional, because it interferes with the right of privacy of contraceptive users.
A
- (B) Under the commerce clause, Congress has the very broad power to regulate interstate commerce. Generally speaking, Congress has the power to regulate any activity (whether carried on in one state or many) that has any appreciable effect—direct or indirect—upon interstate commerce. This is the so-called “affectation doctrine.” Choice (A) is incorrect. Article I, Section 8, reads: “The Congress shall have Power to lay and collect Taxes, Duties, Imposts and Excises, to pay the Debts and provide for the common Defense and General Welfare of the United States.” The Supreme Court has taken the reference to “general welfare” to mean that Congress enjoys great discretion in how it chooses to allocate money for the public [United States v. Butler, 297 U.S. 1 (1936)]. There is no such allocation of money in our example; just a regulation. Choice (C) is incorrect. The 5th Amendment’s takings clause states: “No person. . . shall have property be taken for public use, without just compensation.” In Lucas v. South Carolina Coastal Council, 505 U.S. 1003 (1992), the Court stated that a taking occurs where “regulation denies all economically beneficial or productive use of the land.” Here, the pharmaceutical company can still sell Autonomy to anyone who has a doctor’s prescription. Choice (D) is incorrect. The Supreme Court has stated that there is a fundamental right to contraceptives [Griswoldv. Connecticut, 381 U.S. 479 (1965) and Eisenstadt v. Baird, 405 U.S. 438 (1972)]. For a law to be deemed invalid as violating this fundamental right, there must be a showing that the law infringes on the right. Here, the federal statute does not appear to be an infringement, since anyone with a doctor’s prescription may obtain the contraceptive.
60
Q
- A company created a new brand of pies. However, a study by the federal Food and Drug Administration revealed that the pies contain potentially harmful levels of nuts for some consumers with nut allergies. As a result, Congress enacted legislation prohibiting the shipment and sale of the pies across state lines.A state has a statute that regulates the shipment and sale of the pies within its territory. In light of the federal legislation prohibiting the shipment and sale of the pies across state lines, the state statute is probably(A) constitutional, because it is within the state’s police power.(B) constitutional, because Congress may not regulate an economic activity where both buyer and seller reside in the same state.(C) unconstitutional, because it affects interstate commerce.(D) unconstitutional, because the federal law preempts any conflicting state legislation regarding the sale and shipment of the pies.
A
- (A) In this question, the Vermont statute simply regulates the shipment and sale of the pieswithin the state’s borders (or intrastate). As a result, where Congress has not acted,the states do have police powers to reguLate any phase of local business (production,marketing, sales, etc.) provided that such regulations neither discriminate against norburden interstate commerce [Parker v. Brown, 317 U.S. 341 (1943)] or violate otherprovisions of the Constitution. Choice (B) is incorrect. Congress, using its commerceclause powers under Article I, Section 8, may regulate economic activities where bothbuyer and seller reside in the same state,if such activities have a “substantial effect”on interstate commerce. [Katzenbach v. McClung, 379 U.S. 641 (1966)]. Choice (C) isincorrect. Even if the state regulation affects interstate commerce, the state is entitledto pass such Legislation under its 10th Amendment police powers to regulate health,safety, weLfare, and morals. However, the state regulation may not impose any undueburdens on interstate commerce; in that case, the regulation would be deemed toviolate the dormant commerce clause of Article I, Section 8. Choice (D) is incorrectbecause there is no conflict between the federal law and the state law. Nor is Congressregulating a field, like immigration, that is exclusive to it.
61
Q
- A manufacturer created a new brand of fish tackle which they sold throughout the country. Congress now imposes a tax of 14 cents on each set of tackle sold in the United States.This tax is(A) unconstitutional, because the tackle is sold in interstate commerce.(B) unconstitutional, because it interferes with the sovereign right of state governments to engage in intrastate commerce.(C) constitutional, because the supremacy clause validates laws enacted by Congress.(D) constitutional, because it is within the power of Congress to raise revenue.
A
- (D) Article I, Section 8 provides: “The Congress shall have Power to ay and coLLect Taxes, Duties, Imposts and Excises….” As a generaL rule, if Congress has no power to regulate the activity taxed, the validity of the tax ultimately depends on its validity as a revenue-raising measure. Choice (D) is correct because as long as the dominant intent of the taxis revenue raising, it will be upheld even though the tax may have substantial regulatory effect. Choice (A) is incorrect, as Congress’s ability to tax under the tax and spend clause is not contingent upon whether the taxed object is sold interstate. Choice (B) is incorrect. State governments do have police powers under the 10th Amendment to regulate health, safety, welfare, and morals. Such powers also include the right to regulate intrastate commerce. However, Congress, under its tax and spend powers in Article I, Section 8, can levy taxes on commerce that is intrastate. Choice (C) is incorrect. The Supremacy Clause of Article VI states that the “Constitution, and the Laws of the United States which shall be made in Pursuance thereof, and all Treaties made, or which shall be made, under the Authority of the United States, shall be the supreme Law of the Land.” The Supremacy Clause does not contain any substantive powers for Congress; instead, the clause declares that any state law that conflicts with an otherwise valid federal law will be deemed unconstitutional. Congress’s power to tax derives from the Tax and Spend Clause of Article I, Section 8.
62
Q
- A company created a drug to lower high blood pressure. The President issues an executive order prohibiting the shipment and sale of the drug within the United States. Under this executive order, federal agents are empowered to prosecute all interstate shippers and sellers of the drug. The President claims that he is acting upon conclusive evidence that the drug causes cervical cancer in laboratory monkeys.This executive order is(A) valid, because the President has the authority to ensure that laws are faithfully executed.(B) valid, because the President has the authority to impose economic regulations unless overruled by Congress.(C) invalid, because it is an unauthorized extension of executive power.(D) invalid, because the President does not have the power to regulate interstate commerce.
A
- (D) Article I, Section 8 grants Congress (not the President) the power to regulate interstate commerce. The President does not have any constitutionally delegated legislative power that is inherently law-making in nature. See Youngstown Sheet& Tube v. Sawyer, 343 U.S. 579 (1952), holding invalid a presidential order directing seizure of steel miLls to prevent a threatened strike. Note that choice (D) is more narrowly correct than choice (C) because it addresses executive intrusion in the area of interstate commerce. Choices (A), (B), and (C) are incorrect. In Youngstown Sheet & Tube v. Sawyer, 343 U.S. 579 (1952), the Court stated that the President does not possess any legislative powers; the President may act only if there is explicit authorization in the Constitution or in a federal statute. Here, the President lacks such authorization and, thus, violates the principle of the separation of powers.
63
Q
- A city imposes a municipal excise tax of $200 per year on commercial photographic studios in the city. It also imposes an excise tax of $100 per year on every itinerant commercial photographer for the privilege of using the streets and sidewalks. A credit is allowed against this latter tax for any excise tax paid to the city by the photographer or his employer in respect to the maintenance of a photographic studio in the city.In an action by a studio located in the city challenging the constitutionality of the $200 per year excise tax, the court will most likely declare the tax(A) constitutional, as a nondiscriminatory license tax.(B) constitutional, as within the powers of the state to tax the instruments of interstate commerce.(C) unconstitutional, as an undue burden on interstate commerce.(D) unconstitutional, as a discriminatory tax on the privilege of doing business within the state.
A
- (A) The $200 per year excise tax on the commercial photographic studios operating in the city would be upheld as a valid license tax. Such license taxes (as well as privilege, franchise, and occupation taxes) when applied to local activities—separate from the interstate commerce, of which they are a part—are generally upheld if nondiscriminatory and not unreasonably burdensome in their impact on the interstate commerce involved. Choice (B) is incorrect. Under the commerce clause of Article I, Section 8, only Congress may regulate interstate commerce, including the instruments thereof. A local government lacks constitutional power to regulate such instruments. Choices (C) and (D) are incorrect. Under the dormant commerce clause of Article I, Section 8, local governments like the city may not unduly burden interstate commerce. In our example, the city is not discriminating against out-ofstaters engaged in commerce. Nor does it seem that the city’s benefits from the tax disproportionately hurt out-of-staters.
64
Q
- A city imposes a municipal excise tax of $200 per year on commercial artists’ studios in that city. It also imposes an excise tax of $100 per year on every itinerant commercial figure drawer for the privilege of using the streets and sidewalks. A credit is allowed against this latter tax for any excise tax paid to the city by the drawer or his employer in respect to the maintenance of an artist’s studio in the city.A gallery, located in a neighboring state, has been sending two itinerant artists into the city. Their practice is to draw a picture of a pedestrian, ask him to order a finished painting, and collect a payment of $2.00. The drawing is sent to a studio, which frames it and mails the framed painting to the customer. The neighboring state does not impose a tax on artists’ studios.The gallery challenges the constitutionality of the $100 per year excise tax that is imposed upon its itinerant artists in the city. The court will most likely declare this tax(A) constitutional, as a valid ad valorem tax on interstate commerce.(B) constitutional, since the tax was levied not on the commercial art business as a whole but on the local activity of drawing pictures.(C) unconstitutional, since a state or municipality may not impose a license tax on orders for goods or services to be perfonned in another state.(D) unconstitutional, as a discriminatory privilege tax.
A
- (B) The $100 per year excise tax that is imposed upon the itinerant artists would be upheLd. In Dunbar-StanleyStudios v. Alabama (1969), a similar fixed fee on transient photographers was upheld as constitutional when applied to an out-of-state firm taking photographs in the taxing state, the rationale being that the tax was levied not on the business as a whole but on a local activity of taking photographs, as opposed to their development and processing. Under this rationa’e, choice (B) is correct, and choice (C) is incorrect. Choice (A) is also incorrect because, underthe commerce clause of Article I, Section 8, only Congress may regulate interstate commerce. Therefore, a state lacks authority to tax interstate commerce. Finally, choice (D) is incorrect. Under the dormant commerce clause of Article I, Section 8, local governments like the city may not unduly burden interstate commerce. In our example, the city is not discriminating against out-of-staters engaged in commerce. Nor does it seem that the city’s benefits from the tax disproportionately hurt out-of-staters.
65
Q
- A state imposes a tax on nonresident photographers who operate photographic studios in the state at a rate of 2% of their state-derived income above $12,000. This income tax exempts taxable income earned outside of the state by photographers who live in the state. Moreover, resident-photographers of the state are not taxed on their in-state earned income. A photographer who operates a studio in the state but is a resident of another state, challenges the constitutionality of this tax.Which of the following provisions would furnish the most applicable basis for this constitutional challenge?(A) The equal protection clause of the Fourteenth Amendment.(B) The due process clause of the Fourteenth Amendment.(C) The commerce clause.(D) The privileges and immunities clause of ArticleIv.
A
- (D) Article IV, Section 2, so far as relevant, reads as follows: “The Citizens of each State shall be entitled to all Privileges and Immunities of Citizens in the several States.” It was designed to ensure to a citizen of State A who ventures into State B the same privileges that the citizens of State B enjoy. In line with this underlying purpose, it was long ago decided that one of the privileges that the clause guarantees to citizens of State A is that of doing business in State B on terms of substantial equality with the citizens of that State. Like many other constitutional provisions, the Privileges and Immunities Clause is not an absolute. It does bar, nevertheless, discrimination against citizens of other states where there is no substantial reason for the discrimination beyond the mere fact that they are citizens of other states. Therefore, the state tax on nonresident photographers should properly be declared unconstitutional as violative of the Privileges and Immunities Clause. Choice (A) is incorrect. The equal protection clause provides heightened protection for groups whose legal classifications are suspect (e.g., classifications based on race, ethnicity, nationality, and religion) and for groups whose legal classifications are quasi suspect (e.g., gender, illegitimacy). The photographer may have been the subject of discrimination by the Legislature, but he is not a member of a suspect or quasi-suspect class. Only rational review will be applied, which the state probably will pass with ease. Choice (B) is incorrect. Generally, a property interest is not protected under the 14th Amendment’s Due Process Clause unless there is a reasonable expectation to continued receipt of the benefit [Board of Regents v. Roth, 408 U.S. 564 (1972)]. In our example, no such reasonable expectation to continued tax benefits appears to exist. Choice (C) is incorrect. States and other local governments have no powers under the Commerce Clause of Article I, Section 8.
66
Q
- A state enacts a statute that prohibits “anyone over60 years of age to run for public office.” A state senator has been in office for three terms and wishes to seek re-election. The senator, who is 61, brings suit challenging the constitutionality of the state statute.Which of the following best states the burden of persuasion?(A) Since a fundamental right is involved, the state must show the regulation is necessary to vindicate a compelling government interest.(B) Since no fundamental right is involved, the petitioner must show the age restriction is not rationally related to a legitimate government interest.(C) The state must show the age regulation substantially furthers an important government objective and does not impair the fundamental right to vote.(D) The petitioner must show the statute violates due process by depriving her of the right to be a candidate.
A
- (B) The Constitution contains no express provision that guarantees the right to be a candidate. The states are free, therefore, to create restrictions on the ability to become a candidate. Certainly, states have used several methods to qualify the right to become a candidate. Even though the Supreme Court has not ruled directly on candidacy age restrictions, interestingly enough, this issue was tested on the Multistate exam recently. Choice (B) is correct because in dicta from previously decided cases, the Supreme Court apparently is applying minimal scrutiny to age restrictions. See “Age and Durational Residency Requirements as Qualifications for Candidacy: A VioLation of Equal Protection?,” 1973, U.Ill. Law Review, 161. Choice (A) is incorrect. There is no fundamental right to run for state office. Accordingly, from the perspective of fundamental rights anaLysis, the court will apply rational review. Choice (C) is incorrect because there is no fundamental right to run for state office. Finally, choice (D) is incorrect because the right to run for public office is not a right protected under the due process clause of the 14th Amendment.Exam Tip: This question needs to be distinguished from Question 67. In Question 67, the state enacted a party affiliation statute placing restrictions on independent candidates. States usually impose demonstrated support requirements on independent candidates or minor political parties. Typically, the demonstrated support statute requires independent candidates or minor parties to submit petitions containing a certain number of signatures from qualified voters before they can receive access to the ballot. In such cases, the Supreme Court generally “has stated that the state needs a compelling or overriding interestto justify classifications and restrictions on political association.” Nowak, pg. 891. On the contrary, this Multistate hypo deals with age restrictions on the right of candidacy. With respect to age classifications, the Supreme Court appears to apply the rational-basis test. Although the Supreme Court has not ruled directly on this age candidacy issue, all related cases involving age discrimination have been adjudged under the minimal scrutiny-rational basistest. In Trafeletv. Thompson, 100 S.Ct. 219 (1979), the Supreme Court refused to review a state law imposing a mandatory retirement age for elected state court judges that was challenged as an age classification violative of equal protection.
67
Q
- A state has enacted a party affiliation statute prohibiting a person from being an independent candidate in a general election if she had either (1) registered with a political party during the year prior to the immediately preceding primary, or (2) voted in that primary. The state adopted the so-called “disaffiliation” statute in order to have intraparty feuds resolved in primary elections rather than in the general election. Moreover, the state’s elections director strongly supported the law and argued that it was necessary to avoid voter confusion and to ensure that the general election winner received a majority.A man, who was a registered Democrat in 2008, now wishes to run as an independent candidate in the November, 2009, general election. However, the elections director ruled that his candidacy violated the state’s “disaffiliation” statute and barred him from appearing on the ballot.If the man files suit in federal district court challenging the constitutionality of the state’s election statute, which of the following best states the burden of persuasion?(A) The state must demonstrate that the law is necessary to further an important state interest under equal protection analysis.(B) The state must demonstrate that the law is necessary to further a compelling state interest under fundamental rights analysis.(C) The man must demonstrate that the law is not rationally related to any legitimate state interest under fundamental rights analysis.(D) The man must demonstrate under fundamental rights analysis that the state has less restrictive alternative means available for independent candidates to get a ballot position.
A
- (B) The right to be a candidate is related to the fundamental right to vote. In general, the state’s interest in limiting ballot access is twofold: (1) to reduce voter confusion, and (2) to maximize the probability that the winning candidate will have received a majority of the popular vote. In Storer v. Brown, 415 U.S. 724, (1974), a California provision that prohibited independent candidates from running in the general election if they had eithervoted in the immediately preceding party primary or registered their party affiliation with a qualified party within one year of the primary was upheld. The Court determined that the “disaffiliation” statute furthered the state’s compelling interest in the stability of the political system and its interest in having “intraparty feuds” resolved in primaries rather than in the general election. Based on the Court’s application of the strict scrutinystandard of review, (B) is the correct answer. The Court noted further in Storer that the state must adopt reasonable alternative means for independent candidates and minor political parties to get a ballot position, and the alternative means must not place too heavy a burden on the right to vote and the right to associate. Choice (A) is incorrect. The language of “important” state interest is usually associated with “intermediate review” in equal protection clause analysis. Intermediate review is reserved for legal classifications based on gender and illegitimacy. In our example, there is no reference to either classification. Choice (C) is incorrect because the right to be a candidate is related to the fundamental right to vote [Storer v. Brown, 415 U.S. 724, (1974)]. Therefore, the court will apply strict scrutiny to any law that infringes such a right. Choice (D) is incorrect. The court will likely apply strict scrutiny (see explanation for Choice B). In that case, the state will have the burden to show that no less-restrictive alternatives exist.
68
Q
- A defendant was charged with the crime of rape. The judge denied him bail pursuant to a state law which states that for the crimes of rape, sexual assault on a child, and sexual assault, no person who stands accused thereof shall be entitled to bail prior to a trial in the courts of this state.The defendant was brought to trial and found guilty. After being sentenced to five to ten years in prison, the defendant appealed his conviction to the highest court in the state. The ground for his appeal was an argument that he was denied his right to counsel at the time of his arrest.While his appeal was pending, the defendant filed a civil rights action in federal court against the judge. The defendant claimed that the judge violated his rights under the excessive bail clause of the Eighth Amendment.The federal court should refuse to hear the case, because(A) the federal court would violate the principle of the separation of powers.(B) the issues are not ripe.(C) the case is moot.(D) the issue of bail is capable of repetition, yet evading review.
A
- (C) In Murphy v. Hunt, 71 L Ed 2d 353, 102 5.Ct 1181 (1982), the U.S. Supreme Court held that a petitioner’s claim against Nebraska’s prohibition of pretriaL bail to a person charged with sexual offenses had violated his constitutional rights under the 8th Amendment was moot, since the petitioner-defendant had already been convicted of the offenses. As a general rule, a case becomes “moot” when the issues presented are no longer “live” or the parties lack a legally cognizable interest in the outcome of the case. Choice (A) is incorrect. Article III, Section 2 states that the judicial power of the federal courts “shall extend to all Cases, in Law and Equity, arising under this Constitution, the Laws of the United States…” Issues pertaining to the 8th Amendment are related to the Constitution. Accordingly, if a federal court were prohibited by the Constitution from hearing this case, the prohibition is not attributable to the principle of the separation of powers. Choice (B) is not the strongest choice. In a sense, this is true. However, the issues in the example will never become ripe; they are moot. Hence, choice (C) is the stronger answer. Choice (D) is incorrect because the bail issue is capable of repetition, but the next aggrieved person can, before conviction, present it for review.
69
Q
- In an attempt to promote safe sex a foundation began sending condoms in the mail. Thousands of Americans became incensed and objected to this type of unsolicited advertising. A group of people started a nationwide campaign against the use of condoms. This new organization also started a strong lobbying movement to have Congress pass legislation prohibiting the distribution of condoms by using the U.S. postal system.Assume that the lobbying effort was successful, and Congress passed a law prohibiting any unsolicited advertising for condoms to be distributed through the U.S. postal system. The foundation has challenged the constitutionality of this federal statute. The best argument against the constitutionality of this law is which of the following?(A) The statute is invalid because it violates the First Amendment protection of commercial free speech.(B) The statute is invalid because it unduly burdens interstate commerce.(C) The statute is invalid because it violates the Fifth Amendment right of privacy.(D) The statute is invalid because it violates the equal protection clause of the Fourteenth Amendment.
A
- (A) This is an extremely difficult Constitutional Law Multistate question. Choice (A) is correct because in Bolger v. Young’s Drug Products Corp., 463 U.S. 60 (1983), a law prohibiting the mailing of unsolicited advertisements for contraceptives was held invalid as violating 1st Amendment’s protection of commercial free speech.The Court held that the government’s interest in protecting recipients from mailthey find “offensive” is insubstantial. Choice (B) is wrong: this is not a state law that unduly burdens interstate commerce but rather a federal law. Choice (C) is wrong because the use of contraceptives is a fundamental right under the protected zone of “marital privacy.” There is a subtle distinction, because this question does not dealwith a statute restricting use of contraceptives. On the contrary, the Law restricts “unsolicited advertising” for contraceptives. Choice (B) is incorrect. The Commerce Clause of Article I, Section 8 permits Congress (and Congress alone) to regulate interstate commerce. Thus, Congress cannot be said to present an “undue burden” on interstate commerce; only local governments can do that and, thus, violate the dormant commerce clause of Article I, Section 8. Choice (C) is incorrect because the 5th Amendment’s right of privacy pertains to the right against self-incrimination, not the right to be Left alone suggested in our example. Finally, choice (D) is incorrect. The equal protection clause provides heightened protection for groups whose legal classifications are suspect (e.g., classifications based on race, ethnicity, nationality, and religion) and for groups whose legal classifications are quasi suspect (e.g., gender, illegitimacy). The foundation may have been the subject of discrimination by Congress, but they are not members of a suspect or quasi-suspect class. Only rational review will be applied, which Congress probably will pass with ease.
70
Q
- In recent years there has been much publicity regarding juries approving excessively high multi- million dollar damage awards in personal injury actions. As a result, Congress enacted a statute that limited recovery in personal injury actions filed in state court(s) to $400,000, and punitive action recovery to a maximum of $750,000.A man was injured in an automobile accident when a car driven by a woman drove through a red light and struck his vehicle. The man was paralyzed from the accident and became a paraplegic. The man brought a personal injury action against the woman in state court. The jury returned a verdict on the man’s behalf and awarded him $1,000,000 in damages.After the jury verdict, the woman filed an appeal challenging the amount of the award, claiming that it was excessive and violated the federal guidelines set forth in the statute.The man has filed suit challenging the constitutionality of the statute. The statute should be ruled(A) unconstitutional, because Congress cannot enact legislation involving local matters, such as automobile accidents, unless it involves interstate commerce.(B) unconstitutional, because a limitation on damage awards in tort actions would violate the equal protection clause of the Fourteenth Amendment as applicable to the states by operation of the Fifth Amendment.(C) constitutional, because under Article III, Congress has plenary power to regulate the jurisdiction and scope ofjudicial review of federal and lower state courts.(D) constitutional, because under the supremacy clause, when there is a conflict between federal law and state law, the federal law preempts and takes precedence over the conflicting state law.
A
- (A) Under its commerce power, Congress has plenary power to regulate interstate commerce and commerce with foreign nations. Obviously, the power of Congress to regulate commerce is very broad. It does, however, have limits so as not to obliterate the distinction between what is national and what is local. To be within Congress’s power under the Commerce Clause, a federal law must either (1) regulate the channels of interstate commerce; (2) regulate the instrumentalities of interstate commerce; or (3) regulate activities thathave a substantial effect on interstate commerce. Since the Tort Liability Reform Act does not affect interstate commerce, it will be invalidated as an impermissible intrusion on local affairs. Therefore, choice (A) is correct. Choice (B) is incorrect. The equal protection clause provides heightened protection for groups whose legal classifications are suspect (e.g., classifications based on race, ethnicity, nationality, and religion) and for groups whose legal classifications are quasi suspect (e.g., gender, illegitimacy). People like the defendant may have been the subject of discrimination by Congress, but they are not members of a suspect or quasi-suspect class. Only rational review will be applied, which Congress probably will pass with ease. Choice (C) is incorrect. Article Ill, Section 2 contains an “exceptions clause”: “In all cases affecting Ambassadors, other public Ministers and Consuls, and those in which a State shall be Party, the supreme Court shall have original Jurisdiction. In all the other Cases before mentioned, the supreme Court shall have appellate Jurisdiction, both as to Law and Fact, with such Exceptions, and under such Regulations as the Congress shall make.” Congress may not use its exceptions clause powers to curtail those powers that belong to the judiciary; such abuse would violate the principle of the separation of powers [ExParteMcCardle, 74 U.S. 506 (1869)1. In our example, Congress would appearto limit unduly the court’s discretion regarding what the plaintiff is owed. Choice (D) is incorrect because under the Supremacy Clause of Article VI, a federal law preempts a state law only if the federal law is constitutional. Here, the federal law lacks constitutional basis.